SlideShare a Scribd company logo
1 of 45
Dermatological drugs
1) Which of the following antiviral agents is NOT phosphorylated by viral thymidine kinase?

A. Valacyclovir

B. Gancyclovir

C. CidofovirCorrect Choice

D. Famciclovir

E. Acyclovir
Cidofovir is a nucleotide analogue antiviral agent. It does not require phosphorylation by virus, but
is converted by host cell kinases to a diphosphate


2) Which biologic agent is administered intramuscularly?

A. Infliximab

B. Efalizumab

C. Etanercept

D. AlefaceptCorrect Choice

E. None of the above
Alefacept is given intramuscularly


3) In addition to minocycline, which of the following drugs has been associated with drug-induced
lupus erythematosus-like syndrome?


A. Rifampin

B. Hydralazine Correct Choice

C. Itraconazole

D. Amiodarone

E. Doxycycline
Drugs associated with drug-induced SLE include minocycline, hydralazine, procainamide, isonaizid,
penicillamine, and anti-convulsants


4) One of the main concerns for prescribing azathioprine to a gout patient with a normal level of
thiopurine methyltransferase on chronic allopurinol is:


A. Patient may develp photosensitivity

B. Azathioprine does not work

C. Allopurinol does not work

D. Patient may become pancytopenicCorrect Choice

E. No concerns
Allopurinol inhibits xanthine oxidase, an enzyme that catabolizes azathioprine. In patients
concurrently taking bothe medications, the allopurinol shunts more 6-MP from the xanthine oxidase
catabolic pathway to the hypoxanthine-guanine phosphoribosyltransferase(HGPRT)anabolic
pathway, creating an excess of purine analogs. This in turn may lead to to excessive
immunosuppression and risk of pancytopenia



                                                  1
5) A patient presents with a likely fixed drug eruption. Her medications include glyburide, lisinopril,
hydrochlorthiazide, and aspirin, as well as an over-the-counter laxative. Which is the most likely
culprit?


A. LaxativeCorrect Choice

B. Hydrochlorthiazide

C. Lisinopril

D. Aspirin

E. Glyburide
Phenopthalein, found in laxatives, is a known cause of fixed drug eruptions


6) Fomivirsen is a single-stranded antisense oligonucleotide FDA-approved for the treatment of:

A. HSV infection

B. EBV infection

C. HPV infection

D. CMV infectionCorrect Choice

E. HIV infection
Fomiversen (Vitraene) represents a new class of therapeutic agents known as antisense drugs. It is
indicated for the treatment of CMV retinitis in patients with AIDS. The most common adverse effects
are ocular inflammation and increases in intraocular pressure


7) Which antihistamine has suppressor T-cell inhibitory activity?

A. Promethazine

B. Fexofenadine

C. Doxepin

D. Cromolyn sodium

E. CimetidineCorrect Choice
Cimetidine is an H2 antihistamine that has suppressor T-cell inhibitory activity, by competitively
blocking their H2 receptors. Immunomodulatory effects are useful for treating mucocutaneous
candidiasis, verruca vulgaris, and condyloma acuminata


8) A 59 year-old woman is diagnosed with acute myelogenous leukemia. Induction chemotherapy is
initiated. A few days later the patient develops tender erythematous plaques on her face. What is
the most likely culprit drug?


A. Methotrexate

B. Cyclophosphamide

C. CytarabineCorrect Choice

D. 5-fluorouracil

E. Hydroxyurea




                                                   2
The most likely diagnosis is neutrophilic eccrine hidradenititis. Cytarabine is the usual offending
agent


9) Which drug has mucocutaneous side effects which can include stomatitis, chelitis, lichen planus-
like eruptions, and pityriasis rosea-like eruptions?


A. GoldCorrect Choice

B. Thalidomide

C. Colchicine

D. Potassium iodide

E. None of the above
Mucocutaneous side effects, which are more common with injectable gold, include stomatitis,
cheilitis, lichen planus-like eruptions, and pityriasis rosea-like eruptions


10) Which of the following is not a side effect of bleomycin?

A. Flagellate hyperpigmentation

B. Serpentine supravenous hyperpigmentationCorrect Choice

C. Pulmonary fibrosis

D. Acrosclerosis

E. Erythromelalgia
All of the above are side effects of bleomycin except serpentine supravenous hyperpigmentation.
This is a side-effect of 5-fluorouracil


11) Side effects from this antihistamine include gynecomastia, impotence, and loss of libido:

A. Fexofenadine

B. Promethazine

C. Cyproheptadine

D. CimetidineCorrect Choice

E. Doxepin
Cimetidine, an H2 antihistamine, also competitively inhibits dihydrotestosterone at the androgen
receptor site, with resultant antiandrogen side effects including gynecomastia, impotence, and loss
of libido


12) Which drug is known to cause an SCLE-like eruption?

A. Barbituates

B. Captopril

C. Minocycline

D. Terbinafine Correct Choice

E. Furosemide
SCLE-like reaction are known to occur most likely from glyburide, griseofulvin, hydrochlorothiazide,
penicillamine, piroxicam, and terbinafine




                                                   3
13) Which of the following drugs may cause acute generalized exanthematous pustulosis?

A. Mercury

B. All of these answers are correctCorrect Choice

C. Cefazolin

D. Ampicillin

E. Azithromycin
Acute generalized exanthematous pustulosis (AGEP) has been associated with beta-lactam
antibiotics, macrolide antibiotics, cephalosporins, and mercury.


14) What is the most likely congenital defect associated with isotretinoin therapy?

A. Ventral septal defect

B. Cranial synostosisCorrect Choice

C. Spina bifida

D. Phocomelia

E. Atrial septal defect
Isotretinoin is thought to cause congenital defects by interfering with neural crest development. The
most likely congenital defect is cranial synostosis


15) What drug is known to cause scotomas?

A. Gold

B. Dapsone

C. Quinacrine

D. Isotretinoin

E. Chloroquine Correct Choice
True retinopathy is associated with “bull’s eye” pigment deposition, central scotoma, and diminished
visual acuity. Chloroquine is at greatest risk of causing retinopathy. Quinacrine is not associated
with the risk of retinopathy


16) Finasteride is a specific inhibitor of:

A. Aromatase

B. Type II 5 alpha reductaseCorrect Choice

C. Dihydrotesterone reductase

D. Testosterone synthetase

E. Type I 5 alpha reductase
Finasteride, a type II 5 alpha reductase inhibitor, given as a 1mg tablet daily, is effective in
preventing further hair loss and in increasing the hair counts to the point of cosmetically
appreciatable results in men ages 18 to 41 with mild to moderate hair loss at the vertex, in the
anterior midscalp, and the frontal region




                                                    4
17) Which of the following pairings of antifungal agents and their mechanisms of action is NOT
correct?


A. Fluconazole: Inhibits squalene epoxidaseCorrect Choice

B. Ketoconazole: Blocks conversion of lanosterol to ergosterol

C. Itraconazole: Inhibits 14-alpha-demethylase

D. Griseofulvin: Disrupts microtubule mitotic spindle formation

E. Terbinafine: Inhibits squalene epoxidase
Terbinafine inhibits squalene epoxidase and blocks the biosynthesis of ergosterol, a sterol essential
to the integrity of fungal cell membranes. Itraconazole inhibits 14-alpha-demethylase, blocking
lanosterol conversion to ergosterol. Griseofulvin disrupts microtubule mitotic spindle formation
causing metaphase arrest. Ketoconazole has a mechanism of action similar to itraconazole.
Fluconazole also inhibits 14-alpha-demethylase, not squalene epoxidase


18) Vascular leak syndrome has been associated with which chemotherapeutic agent?

A. Bleomycin

B. Cytarabine

C. Denileukin diftitoxCorrect Choice

D. Methotrexate

E. Interferon
Denileukin difitox (brand name Ontak) is approved for the treatment of cutaneous T-cell lymphoma.
Denileukin difitox is a fusion protein composed from a portion of IL-2 with diphtheria toxin. The
chemotherapy is taken up by cells expressing high-affinity IL-2 receptors. Capillary leak syndrome
presenting with hypotension, edema, pleural effusions, and weight gain caused by fluid retention
has been reported in patients receiving denileukin difitox


19) Methotrexate toxicity with myelosuppression is treated with what agent?

A. Folic acid

B. Vitamin E

C. Folinic acid Correct Choice

D. Cimetidine

E. Oral methylene blue
Leukovorin, or folinic acid, is able to bypass dihydrofolate reductase in the cell division pathway. It
is administered under conditions of methotrexate-induced myelosuppression. Leukovorin is also
available as an oral form, and may be administered as a continual part of methotrexate therapy,
instead of folic acid. It is now believed that neither folic acid nor folinic acid inhibit the efficacy of
methotrexate’s antipsoriatic actions


20) Which one of the following antifungals requires an acidic environment for its absorption?

A. terbinafine

B. fluconazole

C. itraconazoleCorrect Choice




                                                     5
D. griseofulvin

E. amphotericin B
Itraconazole is a triazole whose mechanism, like the other azoles, is inhibition of cytochrome P-450
enzyme lanosterol 14-alpha demethylase, with resultant inhibition of lanosterol conversion to
ergosterol. Ergosterol is an essential component of fungal cell membranes. Itraconazole is a weak
base, which is highly lipophilic and virtually insoluble in water. It is ionized only at a low pH.
Griseofulvin is administered in divided doses for the treatment of tinea capitus. Its absorption is
optimized when given with a fatty food. Oral fluconazole is very well absorbed when given orally
with over 90% bioavailability. Terbinafine, though highly lipophilic, has 70 to 80% absorption, when
administered orally, with a bioavailability not significantly impacted by food intake


21) Bone marrow suppression can occur more often in individuals taking azathioprine with
genetically low allele activity of what enzyme?


A. Dihydrofolate reductase

B. Inosine monophosphate dehydrogenase

C. Glucose-6-phosphatase

D. Epoxide hydroxylase

E. Thiopurine methyltransferase Correct Choice
NEEDS EXPLANATIONS


22) What antifungal is the best choice for a patient with mucocutaneous candidiasis who is currently
taking antacids?


A. Griseofulvin

B. Fluconazole Correct Choice

C. Terbinafine

D. Itraconazole

E. Ketoconazole
Ketoconazole and itraconazole require an acidic environment. Fluconazole, however, does not
require an acidic environment and can work safely and effectively in patients taking antacids, which
can raise gastric pH levels


23) The treatment of choice for scabies in pregnant women is:

A. Malathion

B. Thiabendazole

C. 1% permethrin

D. Precipitated sulfur 6% Correct Choice

E. Ivermectin
NEEDS EXPLANATIONS


24) Approximately what percentage of patients with drug hypersensitivity syndrome will have liver
function test abnormalities?


A. Close to 100%



                                                  6
B. 25%

C. Less than 10%

D. 75%

E. 50% Correct Choice
Drug hypersensitivity syndrome is characterized by fever, skin eruption and internal organ
involvement. Drugs associated with drug hypersensitivity syndrome include sulfonamindes,
dapsone, anticonvulsants (carbamezapine, phenobarbitol, lamotrigine), anti-retrovirals (ritonovir,
nevirapine) and minocycline. Approximately 50% of patients will have abnormal liver enzymes


25) Weekly CD4 T-cell counts are recommended for psoriasis patients treated with which biologic
agent?


A. Etanercept

B. None of the above

C. Efalizumab

D. AlefaceptCorrect Choice

E. Infliximab
Alefacept eliminates activated memory T-cells, so weekly CD4 T-cell counts are recommended


26) The antihistamine with strong H1 and H2 blockade is:

A. Cyproheptadine

B. Cimetidine

C. Cetirizine

D. Doxepin Correct Choice

E. Chlorpheniramine
Doxepin, a tricyclic antidepressant, has H1 and H2 antihistamine activity


27) Which of the following statements regarding retinoids is true?

A. Tretinoin and isotretinoin are second-generation synthetic retinoids

B. Second-generation retinoids are polyaromatic retinoids

C. Bexarotene is a third-generation retinoidCorrect Choice

D. Etretinate is a first-generation retinoid

E. Tazarotene is a fourth-generation selective retinoid
First-generation retinoids are tretinoin (all-trans RA) and isotretinoin (13-cis RA). Second-
generation synthetic retinoids are etretinate, which was replaced by its metabolite acitretin. Third-
generation (polyaromatic retinoids) include the arotinoids, tazarotene, adapalene, and bexarotene


28) Which of the following antifungal agent works by way of inhibiting ergosterol synthesis by
blocking squalene epoxidation:


A. Terbinifine NaftifineCorrect Choice

B. Itraconazole



                                                   7
C. Naftifine

D. Itraconazole and Terbinifine

E. Terbinifine
Terbinafine and Naftifine work by way of inhibiting ergosterol synthesis by blocking squalene
epoxidation (B&C). Itraconazole works by inhibiting ergosterol synthesis by blocking Lanosterol 14-
alpha demthylase


29) At standard dosages, which of the following is fungicidal?

A. TerbinafineCorrect Choice

B. Itraconazole

C. Ketoconazole

D. Griseofulvin

E. Fluconazole
Terbinafine block ergosterol synthesis early in the synthetic pathway by inhibitins squalene
epoxidase. Squalene then accumulates within fungal cells and discupts cell membranes. At standard
dosaging, it is believed to be fungicidal. The other choices are fungistatic


30) Which drug has been associated with cholestatic hepatitis?

A. Clindamycin

B. ErythromycinCorrect Choice

C. Doxycycline

D. Rifampin

E. Ampicillin
The estolate form of erythromycin has been associated with cholestatic hepatitis


31) Clofazamine hyperpigmentation has been described as:

A. Red-brown hyperpigmentation within skin lesions Correct Choice

B. Slate gray-purple hyperpigmentation in a photo-exposed distribution

C. Yellow discoloration of the skin, sclera

D. Dark hyperpigmented streaks in the nails

E. Blue-gray hyperpigmentation over the anterior shins, palate, ears
Clofazamine can induce red-brown hyperpigmentation within skin lesions of patients with Hansen’s
disease


32) Of the new biologic therapies for psoriasis, which agent requires weekly CD4 T-cell count
monitoring?


A. Adalimumab

B. Efalizumab

C. Alefacept Correct Choice




                                                  8
D. Infliximab

E. Etanercept
Alefacept is a human LFA-3/IgG fusion protein, which blocks LFA-3 on antigen presenting cells from
interacting with CD-2 on T-cells, preventing T-cell stimulation. Alefacept also eliminates activated
memory-effector T-cells, so weekly CD4 T-cell counts are required


33) Which drug has recently been shown to be of use in chemoprevention of squamous cell
carcinoma in recessive dystrophic epidermolysis bullosa:


A. Methotrexate

B. Mycophenolate mofetil

C. Cyclosporine

D. IsotretinoinCorrect Choice

E. Topical tar
Despite low therapeutic responses of advanced cancers to retinoids, these drugs appear to have a
promising role in chemoprevention. Patients with oral leukoplakia, actinic keratoses, arsenic
keratoses, and Bowen's disease can benefit from retinoid therapy


34) The mechanism of action of podophyllin most closely resembles that of what other drug listed
below?


A. Dactinomycin

B. Permethrin

C. 5-fluorouracil

D. Chlorambucil

E. Colchicine Correct Choice
Both podophyllin and colchicine have antimitotic activity. They bind to tubulin dimers, interfering
with mitotic spindle and microtubule assembly


35) Raynaud’s phenomenon is a potential adverse effect of what chemotherapeutic agent?

A. Doxorubicin

B. Hydroxyurea

C. Vinca alkaloids

D. 5-fluorouracil

E. Bleomycin Correct Choice
Bleomycin use has been associated with Raynaud’s phenomenon occurring in digits treated with
intralesional therapy for periungual and plantar warts


36) What antihistamine can cause gynecomastia, impotence, and loss of libido?

A. Doxepin

B. Cyproheptadine

C. Cimetidine Correct Choice




                                                  9
D. Fexofendine

E. Chlorpheniramine
Cimetidine competitively inhibits dihydrotestosterone at the androgen receptor site and can exhibit
anti-androgen effects.


37) Methemoglobinemia is an adverse reaction to treatment with which agent?

A. Plaquenil

B. DapsoneCorrect Choice

C. Methotrexate

D. Cyclophosphamide

E. Azathioprine
Dapsone is a lipid-soluble sulfone drug that is used widely in dermatology in a variety of conditions
including dermatitis herpetiformis, leprosy, and neutrophilic dermatoses. Adverse effects from
dapsone are both pharmacologic and idiosyncratic and include hemolytic anemia,
methemoglobinemia, agranulocytosis, hypersensitivity syndrome and neuropathy. Of these, the first
two are pharmacologic and anticipated, to some degree, in most patients treated with dapsone.
However, the magnitude of toxicity varies greatly among individuals on the drug.
Methemoglobinemia is the formation of methemoglobin in the blood, which has a decreased oxygen-
carrying capacity compared with hemoglobin and can result in cyanosis. The reaction is related to
the N-hydroxy metabolites of dapsone, which are potent oxidants. G6PD-deficient individuals are
more susceptible to oxidative stresses, including those from dapsone metabolites, and a baseline
G6PD level is recommended prior to initiation of dapsone therapy. In the event of emergent
methemoglobinemia, oral methylene blue (100-300 mg/day) can be used to decrease
methemoglobin levels. However, if the patient is G6PD deficient, this strategy is ineffective


38) Which of the following statements is NOT true regarding the categories for safety of drug use in
pregnancy?


A. Drugs are category D if controlled studies show risk to human fetus, but in some instances
benefits may outweigh risks

B. Drugs are category A if controlled studies in humans show no risk to the fetus

C. Drugs are category B if controlled human studies show no risk to the fetus but may show risk to
animals, or if no risk has been shown in animal studies but no human studies have been conducted

D. Category X drugs are contraindicated in pregnancy

E. Drugs are category C if risk to the human fetus has been demonstrated, but animal studies are
equivocalCorrect Choice
Drugs are category C if risk to the human fetus cannot be ruled out, studies are lacking, or animal
studies are equivocal. Drugs for which risk to the human fetus has been demonstrated are
pregnancy category D


39) The risk of pseudotumor cerebri in patients taking isotretinoin is increased by:

A. Dehydration

B. Comorbid affective disorder

C. Doses higher than 1.0 mg/kg/day

D. Concomitant use of tetracycline Correct Choice




                                                 10
E. Concomitant use of TMP-SMX
The risk of pseudotumor cerebri is increased in patients on isotretinoin and a tetracycline


40) Neutrophilic eccrine hidradenitis is associated most often with which chemotherapeutic agent?

A. Vinca alkaloids

B. 5-fluorouracil

C. Cytarabine Correct Choice

D. Dactinomycin

E. Doxorubicin
Neutrophilic eccrine hidradenitis occurs in neutropenic patients with malignancies, usually acute
myelogenous leukemia. Cytarabine is the most commonly associated medication


41) Which of the following statements regarding antifungal medications is TRUE?

A. Terbinafine is fungistatic

B. Griseofulvin is safe for patients with variegate porphyria

C. Griseofulvin is a cytochrome P450 3A4 inhibitor

D. Ketoconazole is fungicidal

E. Ketoconazole has been associated with gynecomastiaCorrect Choice
Medications that can precipitate acute attacks in variegate porphyria include barbiturates, estrogen,
griseofulvin, sulfonamides and ethanol. Ketoconazole can produce impotence and gynecomastia by
interfering with androgen synthesis. Ketoconazole inhibits cytochrome (CYP) P450 and most
concerning, can rarely cause fulminant hepatitis. Griseofulvin induces CYP P450, not inhibits it.
Terbinafine is fungicidal along with amphotericin B. Ketoconazole is fungistatic


42) For which of the following medications is abdominal cramping and watery diarrhea a very
common side effect that may limit treatment?


A. ColchicineCorrect Choice

B. Chlorambucil

C. Potassium iodide

D. Gold

E. Thalidomide
Colchicine is an alkaloid with antimitotic activity that is used in dermatology for its effects on
neutrophils. The most common side effect from colchicine use is gastrointestinal distress with
abdominal cramping and watery diarrhea


43) The combination of ethanol and acitretin is potentially problematic because:

A. Ethanol inhibits the cytochrome p450 system

B. Ethanol exacerbates the cheilitis caused by acitretin

C. Ethanol promotes the conversion of acitretin to etretinateCorrect Choice

D. Ethanol promotes the metabolism of acitretin




                                                   11
E. Acitretin increases the toxicity of ethanol
Acitretin is an synthetic retinoid with affinity to the retinoic acid receptor (RAR). It can modulate the
proliferation and differentiation of epidermal keratinocytes. The concurrent injestion of ethanol and
alcohol increases the formation of etretinate. Unlike acitretin, etretinate is more lipophilic and
accumulates in the fat. It therefore has a much longer elimination half-life, estiimated at 120 days


44) The combination of ethanol and acitretin is potentially problematic because:

A. Ethanol inhibits the cytochrome p450 system

B. Ethanol exacerbates the cheilitis caused by acitretin

C. Ethanol promotes the conversion of acitretin to etretinateCorrect Choice

D. Ethanol promotes the metabolism of acitretin

E. Acitretin increases the toxicity of ethanol
Acitretin is an synthetic retinoid with affinity to the retinoic acid receptor (RAR). It can modulate the
proliferation and differentiation of epidermal keratinocytes. The concurrent injestion of ethanol and
alcohol increases the formation of etretinate. Unlike acitretin, etretinate is more lipophilic and
accumulates in the fat. It therefore has a much longer elimination half-life, estiimated at 120 days


45) Which of the following supplements is most likely to decrease hemolysis associated in patients
taking dapsone?


A. Vitamin A

B. Folic acid

C. Vitamin ECorrect Choice

D. Vitamin B6

E. Vitamin D
Adverse effects from dapsone are both pharmacologic and idiosyncratic and include hemolytic
anemia, methemoglobinemia, agranulocytosis, hypersensitivity syndrome and neuropathy. Of these,
the first two are pharmacologic and anticipated, to some degree, in most patients treated with
dapsone. However, the magnitude of toxicity varies greatly among individuals on the drug.
Methemoglobinemia is the formation of methemoglobin in the blood, which has a decreased oxygen-
carrying capacity compared with hemoglobin and can result in cyanosis. The reaction is related to
the N-hydroxy metabolites of dapsone, which are potent oxidants. G6PD-deficient individuals are
more susceptible to oxidative stresses, including those from dapsone metabolites, and a baseline
G6PD level is recommended prior to initiation of dapsone therapy. Vitamin E (800 IU/day) has been
shown to provide a small amount of protection against methemoglobinemia and hemolysis,
however, the clinical benefit of this strategy is unclear


46) Which of the following has been associated with a lichenoid drug eruption?

A. Acetaminophen

B. Sulfasalazine

C. Hydrochlorothiazide Correct Choice

D. Erythromycin

E. Nicotinamide
Lichen-planus-like (lichenoid) drug eruptions have been reported with: antimalarials, ?-blockers,
captopril, gold, penicillamine, HCTZ, NSAIDs. Lichenoid drug reactions are often photodistributed




                                                   12
47) Which of the following is a side effect of hydroquinone cream?

A. Telangiectasia

B. Photosensitivity

C. Atrophy

D. OchronosisCorrect Choice

E. Tachyphylaxis
Exogenous ochronosis is an uncommon complication of irreversible pigmentation due to overuse of
topical hydroquinone (1,4 dihydroxybenzene). Hydroquinone acts to by melanocyte pigment
production by auto-oxidation of melanin, tyrosinase and phenol oxidases.


48) Which antiparasitic agent is highly flammable?

A. MalathionCorrect Choice

B. Lindane

C. Precipitated sulfur

D. Thiabendazole

E. Permethrin
Malathion, an organophosphate cholinesterase inhibitor used to treat scabies and head lice, is
flammable


49) Cyclosporine forms a complex to directly interfere with activation of what calcium-dependent
protein?


A. Calcineurin Correct Choice

B. Cyclophilin

C. Cytochrome P-450

D. Calmodulin

E. NFAT-1
Cyclosporine inhibits calcineurin, a phosphatase activated in the presence of calmodulin and
calcium, by cyclophilin. Cyclosporine forms a complex with cyclophilin, blocking its ability to activate
calcineurin, and thus preventing calcineurin from phosphorylating NFAT-1, a transcription factor.
NFAT-1, when phosphorylated can travel to the nucleus of cells, initiate IL-2 production, and
stimulate T-cell proliferation


50) What is the target antigen for rituximab?

A. CD4

B. CD22

C. CD7

D. CD20Correct Choice

E. CD8
Rituximab is an anti-CD20 monoclonal antibody. CD20 is a B-cell marker and is used to treat Non-
Hodgkins B-cell lymphoma. It is also approved for the treatment of rheumatoid arthritis




                                                  13
51) What medication's mechanism of action is via suppression of the halide-myeloperoxidase
system?


A. Hydroxyurea

B. Melphalan

C. Quinicrine

D. DapsoneCorrect Choice

E. Cyclophosphamide
Dapsone inhibits the neutrophil halide-myeloperoxidase system which results in an impaired
respiratory burst and subsequent tissue damage


52) A 56 year-old man presents with blue-gray discoloration on his face, ears, and dorsal hands.
What is the most likely offending agent?


A. Clofazimine

B. Quinacrine

C. Minocycline

D. AmiodaroneCorrect Choice

E. Chloroquine
The patient presents with blue-gray discoloration in sun-exposed areas. The most likely offending
agent is amiodarone. Blue-gray discoloration from minocycline usually occurs on legs. Chloroquine
usually causes blue-gray discoloration in the sclerae, teeth, buccal mucosa, nail beds, and pretibial
areas. Quinacrine causes yellow discoloration of skin and conjunctiva. CLofazimine usualy causes a
red-brown discoloration


53) Painful periungual pyogenic granulomas have been associated with what medication?

A. Indinavir Correct Choice

B. Ketoconazole

C. Doxycycline

D. Valacyclovir

E. Tazarotene
Periungual pyogenic granulomas and painful paronychial eruptions have been reported in
association with various anti-HIV medications including, indinavir, zidovudine, and lamivudine


54) Which drug may increase levels of digoxin?

A. Amoxicillin

B. Minocycline

C. Cephalexin

D. Ciprofloxacin

E. ErythromycinCorrect Choice




                                                 14
Erythromycin inhibits the cytochrome P-450 system, which may result in increased levels of digoxin,
among many other drugs


55) A 10 year old child with a seizure disorder develops a morbiliform eruption and elevated LFT’s
two weeks after starting Dilantin therapy. As his physician you:


A. Discontinue Dilantin and begin Phenobarbital

B. Continue Dilantin and treat rash with topical corticosteroids

C. Discontinue Dilantin and begin carbamazepine

D. Restart Dilantin once the rash resolves

E. Discontinue Dilantin and begin valproic acidCorrect Choice
Anticonvulsant hypersensitivity syndrome (also drug rash with eosinophilia and systemic symptoms
(DRESS0 and dilantin hypersensitivity syndrome) presents with cutaneous eruption accompanied by
fever, facial edema, lymphadenopathy, leukocytosis and hepatitis. Cross reactivity is present in all
aromatic anticonvulsants including phenytoin, carbamazepine and phenobarbitol. There is no cross
reaction with valprioc acid


56) Potentially fatal ventricular arrhythmias can occur with concomitant use of cisapride and:

A. Terbinafine

B. Atorvostatin

C. Astemizole

D. Erythromycin Correct Choice

E. Digoxin
Co-administration of erythromycin with the antihistamines terfenedine and astemizole or the
gastrointestinal promobility agent cisapride increases the risk of torsade de pointes and is
contraindicated. These drugs are no longer available in the US


57) The agent of choice used to acutely lower methemoglobin levels in patients taking dapsone is:

A. Cimetidine

B. Vitamin E

C. Oral methylene blue Correct Choice

D. Homocysteine

E. Glucose-6-phosphatase
Cimetidine and vitamin E have both been known to provide prophylaxis against methemoglobin
formation. G6PD-deficient individuals are at greater risk of hematologic toxicity from dapsone


58) The most specific marker of drug-induced lupus is:

A. Anti-La Ab

B. ANA

C. Anti-ds DNA Ab

D. Anti-histone Ab Correct Choice




                                                  15
E. Anti-Ro Ab
Anti-histone Ab is most specific for drug-induced lupus. In addition to minocycline, hydralazine,
procainamide, isonaizid (INH), penicillamine and anti-convulsants have been associated with drug-
induced lupus-like syndrome


59) Which of the following is the most common adverse effect of Thalidomide therapy?

A. Diarrhea

B. Hypertension

C. Photosensitivity

D. Skin discoloration

E. SedationCorrect Choice
Thalidomide was introduced in the late 1950's as a "safe" sleeping aide. It readily penetrates the
CNS, where it exerts a hyposedative effect comparable with barbiturates. By far, the most common
adverse effect from thalidomide is sedation, which in many patients may require that primarily
night-time doses be utilized


60) Which of the following antiparasitic agents is an organophosphate cholinesterase inhibitor?

A. Ivermectin

B. Precipitated sulfur

C. Thiabendazole

D. Lindane

E. MalathionCorrect Choice
Malathion is an organophosphate cholinesterase inhibitor


61) The treatment of choice for Wegner’s granulomatosis is:

A. Methotrexate

B. Chlorambucil

C. Cyclophosphamide Correct Choice

D. Azaithioprine

E. Systemic glucocorticosteroids
NEEDS EXPLANATIONS


62) Thryoid function tests should be checked before and during therapy with which of the following
medications?


A. Azathioprine

B. Potassium iodideCorrect Choice

C. Thalidomide

D. Colchicine

E. Gold




                                                16
The Wolff-Chaifkoff effect, which is the inhibition of thyroid hormone synthesis from excess iodides
which block organic iodides from binding in the thyroid, can be observed in patients on potassium
iodide therapy. In patients with normal thyroid function, autoregulatory mechanisms allow for
appropriate escape from this effect. In patients with impaired autoregulatory mechanisms, the
Wolff-Chaikoff effect can lead to hypothyroidism


63) Which of the following is a low sedation metabolite of hydroxyzine?

A. Fexofenadine

B. Cyproheptadine

C. Loratadine

D. CetirizineCorrect Choice

E. Ranitidine
Cetirizine is a second-generation H1 antihistamine that is a low sedation metabolite of hydroxyzine


64) Which of the following statements is not true regarding the tetracycline antiobiotics?

A. These antibiotics are effective against Mycoplasma infections

B. Tetracycline is more phototoxic than demeclocyclineCorrect Choice

C. Tetracyclines are contraindicated in children less than 9 years of age

D. Ingestion of zinc salts may impair absorption of tetracycline

E. Tetracycline is the most common cause of fixed drug eruption
Demeclocycline and doxycycline are the most phototoxic of all the tetracyclines


65) Which of the following is not an ingredient of Castellani's paint?

A. Boric Acid

B. Resorcinol

C. Phenol

D. Ethyl acetateCorrect Choice

E. Industrial methylated spirit
Castellani's paint was named after Sir Aldo Castellani and contains resorcinol, acetone, magenta,
phenol, boric acid, industrial methylated spirit, and water. It is fungicidal and bactericidal with local
anesthetic effects. It has been used to treat inflammatory tinea cruris, leg ulcers, and acute
paronychia


66) What is the difference between podophyllin and podophyllotoxin?

A. Essentially the same; they are interchangable in terms of treatment and side effects

B. Podophyllin contains kaempherol which is a potent mutagensCorrect Choice

C. Podophyllotoxin is a phosphodiesterase inhibitor

D. Podophyllotoxin contains quercetin which is a potent mutagens

E. Podophyllin reversibly binds tubulin inhibiting cells in metaphase




                                                   17
Podophyllotoxin, also known as podofilox or Condolox, is a anti-mitotic agent that reversibly binds
tubulin, arresting cells in metaphase. It is used topically to treat genital warts. Podophyllin, which
has the same mechanism of action, contains kaempero and quercetin which are potent mutagens.
Both are derived from the May Apple plant. Cantharin, an antiviral agent derived from the Blister
beetle, is a phosphodiesterase inhibitior


67) Which of the following agents is NOT a UVA blocker?

A. Dioxybenzone

B. Avobenzone

C. Red veterinary petrolatum

D. Amyl p-dimethylaminobenzoateCorrect Choice

E. Dibenzoylmethane
Dioxybenzone is one of the benzophenones (as in oxybenzone and sulisobenzone), which are UVA
blockers. Amyl p-dimethylaminobenzoate is a UVB blocker. Dibenzoylmethane (avobenzone; Parsol
1789) is a UVA/UVB blocker. Red veterinary petrolatum is a UVA blocker


68) Which of the following class of medications has been associated with acquired brachial
dyschromatosis?


A. Anti-virals

B. HMG-CoA Reductase inhibitors

C. Non-Steroidal Anti-inflammatory medications

D. ACE-inhibitorsCorrect Choice

E. Protease inhibitors
Acquired brachial dyschromatosis is a condition described as asymptomatic, gray-brown patches
with geographic borders, occasionally interspersed with hypopigmented macules, on the dorsum of
the forearms, mostly bilaterally and seen in middle aged women. Epidermal atrophy, basal layer
hyperpigmentation, elastosis and angiectases were histopathologic features. An association with
Civatte's poikiloderma as well as hypertension and/or antihypertensive drugs, especially ACE-
inhibitors, is suggested


69) What is used to reduce bladder toxicity from cyclophosphamide?

A. Leukovorin

B. Cimetidine

C. Folic acid

D. Mesna Correct Choice

E. Vitamin E
Mesna or sodium 2-mercptoethanesulfonate, has been used to reduce bladder toxicity from
cyclophophamide


70) The medication most acceptable for usage in patients with renal failure is:

A. Oxytetracycline

B. Tetracycline




                                                  18
C. Minocycline

D. Demeclocycline

E. Doxycycline Correct Choice
Renal failure may prolong the half-life of most tetracyclines except doxycycline. Doxycycline is
excreted via the GI tract, unlike the other tetracyclines


71) Which of the following statements regarding sunscreens is true?

A. Photoallergy has not been reported to benzophenones

B. Methyl anthranilate is a UVB absorber

C. PABA and its derivates do not cross react with sulfonamides

D. Padimate O is a UVB absorberCorrect Choice

E. Physical blockers absorb ultraviolet light and convert it to lower energy wavelengths
Physical blockers reflect and scatter UV rays, whereas chemical sunscreens absorb UV light and
convert the absorbed energy into longer lower energy wavelengths. Methyl anthranilate is a UVA
blocker. Padimate O, a PABA derivative, is a UVB blocker. Photoallergy has been reported with
increasing frequency to benzophenones. Allergic contact allergy can occur with PABA and its
derivatives, which can cross react with azodyes, aniline, procaine, benzocaine,
paraphenylenediamine, and sulfonamides


72) A patient with acute diarrhea is prescribed antibiotic treatment for his symptoms. He
subsequently suffers from nausea and vomiting after ingesting alcohol. What is the most likely
medication he is taking?


A. Azithromycin

B. Clindamycin

C. MetronidazoleCorrect Choice

D. Ciprofloxacin

E. Penicillin
This patient is most likely taking metronidazole for acute diarrhea secondary to giardella.
Metronidazole causes antabuse-like reactions with ingestion of alcohol


73) What is the half-life of isotretinoin?

A. 50 hours

B. 20 hoursCorrect Choice

C. 30 days

D. 7 hours

E. 120 days
The half-life of isotretinoin is 20 hours. The half lives of bexarotene, acitretin, etretinate are 7
hours, 50 hours, 120 days respectively


74) Which of the following systemic agents has been shown to be the most effective in the
treatment of toenail onychomycosis?




                                                    19
A. Itraconazole

B. Griseofulvin

C. Ketoconazole

D. TerbinafineCorrect Choice

E. Fluconazole
Craford et al. reviewed the available literature examining the efficacy of systemic anti-fungals and
performed a meta-analysis. Pooled analysis of cure rates at 11 and 12 months suggested that
terbinafine was more effective than itraconazole


75) What is the recommended period for contraception after cessation of acitretin therapy in the
United States?


A. 2 years

B. 3 months

C. 3 years Correct Choice

D. 1 month

E. 1 year
Etretinate has a prolonged half-life of 80-160 days. The levels may persist up to 3 years in the
body. Acitretin can be converted to etretinate in the presence of ethanol


76) The SPF of a sunscreen is based on applying the sunscreen at what concentration?

A. 5 mg/cm2

B. 1 mg/cm2

C. 2 mg/cm2Correct Choice

D. 4 mg/cm2

E. 3 mg/cm2
A sunscreen SPF is based on using it at a concentration of 2 mg/cm2 which is about 1 ounce or 30
grams for the entire average sized body. It also is about 3-5 grams for the head and neck


77) How long after isotretinoin therapy can one safely begin trying to conceive?

A. Two weeks

B. One year

C. Three years

D. Immediately

E. One monthCorrect Choice
A woman should wait one month before trying to conceive after taking isotretinoin to prevent birth
defects. After taking acitretin a woman should wait three years before trying to conceive


78) The following drugs have been implicated in drug-induced subacute cutaneous lupus
erythematosus:


A. Pravastatin



                                                 20
B. Terbinafine

C. None of the above are correct

D. Verapamil

E. All the above are correctCorrect Choice
All of the above choices have been implicated in drug-induced subacute cutaneous lupus
erythematosus


79) Which one of the following sunscreens has an absorption spectrum primarily in the UVA range?

A. Cinnamates

B. PABA (para-aminobenzoic acid)

C. Padimate O

D. Octyl Salicylate

E. Parsol 1789 (butyl dibenzoylmethane)Correct Choice
Avobenzone (Parsol 1789) is primarily a UVA blocker. Photostability of avobenzone may be a
problem if it is combined with octyl methoxycinnamate. Salicylates, PABA, Padimate O, and
cinnamates are primarily UVB blockers


80) What is the half-life of isotretinoin?

A. 50 hours

B. 100 days

C. 10 hours

D. 100 hours

E. 20 hoursCorrect Choice
The half-life of isotretinoin is 20 hours


81) Tazarotene is what category for safety in pregnancy?

A. Category X Correct Choice

B. Category A

C. Category C

D. Category B

E. Category D
Category X drugs include: acitretin, etretinate, estrogens, finasteride, 5-fluorouracil, flutamide,
isotretinoin, methotrexate, stanozolol, thalidomide, and tazarotene


82) Which antiviral agent has been associated with fatal thrombotic thrombocytopenic purpura in
AIDS and transplant patients taking high doses?


A. Cidofovir

B. Penciclovir

C. ValacyclovirCorrect Choice




                                                  21
D. Acyclovir

E. Famciclovir
Valacyclovir has been associated with severe and even fatal cases of thrombotic thrombocytopenic
purpura / HUS syndrome in AIDS and transplant patients taking high doses


83) All of the following statements are true regarding cyclosporin A EXCEPT:

A. Adverse effects include hypertrichosis and gingival hyperplasia

B. NSAIDs can potentiate renal toxicity when combined with cyclosporine

C. The most common eletrolyte abnormalities are hypokalemia and hypermagnesemiaCorrect Choice

D. Forms a complex with cyclophilin, blocking its ability to activate calcineurin, thus preventing
calcineurin from phosphorylating NFAT-1

E. Metabolized by the hepatic cytochrome P-450 3A4 enzyme system
The most common electrolyte abnormalities associated with cyclosporin A are hyperkalemia,
hyperuricemia, and hypomagnesemia


84) Which of the following is a true statement about infliximab?

A. FDA-approved for the treatment of psoriasis

B. Not effective in psoriatic arthritis

C. Increases C-reactive protein

D. Chimeric monoclonal antibody against TNF-alphaCorrect Choice

E. Indicated for mild to moderate psoriasis
Infliximab (brand name Remicade) is a chimeric recombinant fusion protein composed of a human
TNF-alpha receptor with IgG1-Fc. The immunoglobulin portion is derived from a human constant
region and a murine variable region. Infliximab neutralizes both soluble and transmembrane TNF-
alpha. This medication has been FDA approved for the treatment of Crohn’s disease, rheumatoid
arthritis and ankylosing spondylitis


85) Which of the following drugs is correctly matched to its target enzymes?

A. Terbinafine-Cytochrome p450

B. Acyclovir-DNA polymeraseCorrect Choice

C. Mycophenolate Mofetil-Phospholipase A2

D. Tacrolimus-Thymidine kinase

E. Methotrexate-Inosine monophosphate dehydrogenase
Drug/Specific enzyme inhibited
Tacrolimus - Calcineurin
Methotrexate - Dihydrofolate reductase
Mycophenolate mofetil - Inosine monophosphate dyhydogenase
Acyclovir - DNA Polymerase
Terbinafine - Squalene epoxidase


86) The mechanism of action of mycophenolate mofetil most closely resembles that of what other
drug?




                                                  22
A. Hydroxyurea

B. Chlorambucil

C. Doxorubicin

D. Azathioprine Correct Choice

E. Cyclophosphamide
Both azaithioprine and mycophenolate mofetil directly interfere with purine synthesis


87) Which biologic agent blocks T-cells from egressing the vasculature and entering the skin?

A. EfalizumabCorrect Choice

B. Alefacept

C. Infliximab

D. Etanercept

E. None of the above
Efalizumab blocks LFA-1 on T-cells from interacting with ICAM-1 on antigen presenting cells,
endothelial cells, and cells in the dermis and epidermis. It thus blocks T-cells from egressing the
vasculature and entering the skin


88) Which of the following chemotherapeutic agents causes increased growth of eyelashes?

A. Mitomycin

B. Methotrexate

C. Cytarabine

D. Interleukin 2

E. Interferon alphaCorrect Choice
Interferons can cause increased growth of eyelashes. Trichomegaly has been reported after
treatment with interferon-alpha in patients with chronic hepatitis, B-cell lymphoma, chronic
granulocytic leukemia, and cutaneous melanoma. Trichomegaly has also been reported in
associatoin with latanoprost, minoxidil, cyclosporine, phenytoin, psoralen, and penicillamine


89) For which of the following medications is sedation a very common side effect that may limit
treatment?


A. Gold

B. Colchicine

C. Chlorambucil

D. ThalidomideCorrect Choice

E. Potassium iodide
Sedation is a very common side effect of treatment with thalidomide. It is additive with other
sedatives, such as alcohol and barbiturates


90) non-pigmenting fixed drug eruption is known to be caused by what agent?

A. Phenopthalein



                                                  23
B. Naproxen

C. Tetracycline

D. Barbiturates

E. Pseudoephedrine Correct Choice
All agents listed are associated with fixed drug eruptions, however, pseudoephedrine hydrochloride
is the one most commonly associated with non-pigmenting fixed drug eruptions


91) The Wolff-Chaikoff effect is associated with what medication?

A. Potassium iodide Correct Choice

B. Zidovudine

C. Bexarotene

D. Thalidomide

E. Hydroxychloroquine
The Wolff-Chaikoff effect is the inhibition of thyroid hormone synthesis from excess iodides which
block organic iodides from binding in the thyroid. In patients with normal thyroid function,
autoregulatory mechanisms allow for escape from this effect. In patients with impaired
autoregulatory mechanisms, the Wolff-Chaikoff effect can lead to hypothyroidism. Thyroid function
should be evaluated and monitored with patients started on potassium iodide


92) Which drug has been associated with an increased incidence of serum sickness in children?

A. CefaclorCorrect Choice

B. Clarithromycin

C. Ciprofloxacin

D. Clindamycin

E. Rifampin
Cefaclor has been associated with an increased incidence of serum sickness in children; the other
drugs have not


93) Combination oral contraceptives decrease free testosterone levels by:

A. Acting as competitive inhibitors of the androgen receptor

B. Directly binding free testosterone

C. Acting as a GnRH antagonist

D. Acting as a GnRH agonist

E. Increasing SHBG (sex hormone binding globulin) production Correct Choice
Oral contraceptives decrease free testosterone levels by increasing the production of sex hormone
binding globulin (SHBG).


94) Thalidomide is most associated with what adverse effect?

A. Distal motor neuropathy

B. Sensory neuropathy Correct Choice



                                                 24
C. Oral ulceration

D. Hypothyroidism

E. Photosensitivity
The most common presentation of the neuropathy from thalidomide is a mild proximal muscle
weakness with symmetric painful paresthesias of the distal extremities with accompanying lower
limb sensory loss. Hypothyroidism is a rarely reported adverse effect.


95) The mechanism of action of efalizumab in the treatment of psoriasis is:

A. Inhibition of p-selectin

B. Inhibition of tumor necrosis factor alpha

C. Inhibition of T cell trafficking into the skin Correct Choice

D. Inhibition of e-selectin

E. Inhibition of macrophage maturation
Efalizumab is a humanized monoclonal antibody directed against CD11a, a component of LFA1.
Efalizumab blocks both T cell activation and trafficking of T cells to the skin. It is given once weekly
as a subcutaneous injection. It’s side effects include rare thrombocytopenia and occasional rebound
of psoriasis upon its abrupt discontinuation


96) All of the following retinoids are excreted in the urine EXCEPT:

A. BexaroteneCorrect Choice

B. Etretinate

C. Isotretinoin

D. Acitretin

E. Tretinoin
Bexarotene is excreted via hepatobiliary excretion. The others are excreted in bile and urine


97) The steroid with the least minerocorticoid activity is:

A. Hydrocortisone

B. Prednisone

C. Methylprednisolone Correct Choice

D. Cortisone

E. Prednisolone
Of the corticosteroids listed, the steroid with the lowest mineralcorticoid activity is
methylprednisolone. Minerocorticoids act on the kidney to decrease the rate of sodium excretion
(with accompanying retention of water). Triamcinolone, dexamethasone, and betamethasone also
have low mineralcorticoid activity
The steroid with the least minerocorticoid activity is:


A. Hydrocortisone

B. Prednisone

C. Methylprednisolone Correct Choice




                                                    25
D. Cortisone

E. Prednisolone
Of the corticosteroids listed, the steroid with the lowest mineralcorticoid activity is
methylprednisolone. Minerocorticoids act on the kidney to decrease the rate of sodium excretion
(with accompanying retention of water). Triamcinolone, dexamethasone, and betamethasone also
have low mineralcorticoid activity


98) The most common side effect of treatment with interferon-alpha is:

A. Liver toxicity

B. Flu-like symptomsCorrect Choice

C. Spastic diplegia

D. Weight loss

E. Nausea
The most common side effect of treatment with interferon-alpha is flu-like symptoms of fever, chills,
myalgias, headache and arthralgias. Prophylactic administration of non steroidal anti-inflammatory
medications may alleviate some of these symptoms


99) Which of the following medications is most likely to interefere with the efficacy of oral
contraceptives?


A. RifampinCorrect Choice

B. Doxycycline

C. Tetracycline

D. Amoxicillin

E. Trimethoprim-sulfamethaxasole
Rifampin has been shown to decrease the efficacy of oral contraceptives. It is an inducer of
cytochrome p450 which increases the metabolism of hormones thereby decreasing the efficacy of
oral contraceptives. There is no clear decrease in oral contraceptive efficacy with concomitant use of
ampicillin, ciprofloxacin, clarithromycin, doxycyline, metronidzole, ofloxacin, or tetracycline


100) Which chemical sunscreen has UVB and UVA II absorption capability?

A. Titanium dioxide

B. Oxybenzone Correct Choice

C. Methyl anthranilate

D. Padimate O

E. Octyl salicylate
The benzophenones, oxybenzone and dioxybenzone, have the broadest absorption spectrum of the
chemical sunscreens, with UVB and UVA II range. Methyl anthranilate, octyl salicylate, and
padimate O are UVB-absorbing chemicals. Titanium dioxide is not a chemical absorber, it is a
physical blocker


101) Alternate-day administration of oral steroids can reduce all of the following side effects
except?




                                                  26
A. Growth impairment

B. Cataracts Correct Choice

C. HPA axis suppression

D. Peptic ulcer disease

E. Opportunisitic infection
Alternate-day corticosteroid dosing regimens does not decrease the risks of posterior subcapsular
cataracts, osteoporosis, and possibly osteonecrosis


102) Which of the following medications can lead to hematologic toxicity when combined with
methotrexate?


A. Dapsone

B. None of these answers are correct

C. Sulfonamides

D. All of these answers are correctCorrect Choice

E. Trimethoprim
All of the above inhibit the folic acid metabolic pathway, and can lead to hematologic toxicity when
combined with methotrexate


103) Terbinafine exerts its antifungal activity by what manner?

A. Interference with cell respiratory processes

B. Inhibition of 14-a demethylase

C. Inhibition of epoxide hydroxylase

D. Inhibition of squalene epoxidase Correct Choice

E. Direct binding to membrane sterols, increasing permeability
Terbinafine, an allylamine, interferes with ergosterol synthesis by inhibiting squalene epoxidase. The
azoles inhibit 14-a demethylase. Nystatin is a polyene which binds irreversibly to membrane sterols,
resulting in a permeability shift. Ciclopirox does not appear to affect sterol biosynthesis but instead
interferes with cell respiratory processes


104) What antifungal is known to cause gynecomastia and impotence?

A. Ketoconazole Correct Choice

B. Terbinafine

C. Griseofulvin

D. Itraconazole

E. Fluconazole
Ketoconazole is known to cause gynecomastia and impotence, by interfering with androgen and
glucocorticoid synthesis


105) All of the following agents exert their function in a cell-cycle specific manner except:

A. Hydroxyurea



                                                    27
B. Methotrexate

C. Azathioprine

D. Cyclophosphamide Correct Choice

E. 5-fluorouracil
Cyclophosphamide is a cell-cycle nonspecific agent, which produces DNA cross-linkages at any point
in the cell cycle. Methotrexate, azaithioprine, and hydroxyurea are S-phase specific cytotoxic
agents. 5-fluorouracil is a cell-cycle specific pyrimidine antagonist


106) The t 1/2 of isotretinoin is:

A. 7 hours

B. 2 days

C. 1 hour

D. 120 days

E. 20 hours Correct Choice
The t 1/2 of isotretinoin is 20 hours. The other answers list the t 1/2 times of various retinoids


107) All of the following topical antioxidants have demonstrated cutaneous anticarcinogenic effects
in mice except:


A. ZincCorrect Choice

B. Silymarin

C. Vitamin E

D. Vitamin C

E. Tea polyphenois
Anti-oxidants are thought to be protective against photoinjury by neutralizing oxygen radicals.
Vitamin C, Vitamin E, tea polyphenois, and silymarin are all anti-oxidants


108) A 15 year old boy presents with a 4 month history of pigmented bands on several fingernails
and toenails. The most like etiology is:


A. Nevomelanocytic nevi

B. Peutz-Jeghers syndrome

C. Acral lentiginous melanoma

D. Chloroquine therapy

E. Minocycline therapy Correct Choice
Melanonychia occurring simultaneously on several nails is most likely to be due to minocycline
therapy. Blue-black pigmentation may be present in nails, skin, scars and sclerae


109) What is the treatment of choice for methemoglobinemia?

A. Observation

B. Aspirin




                                                  28
C. Methylene blueCorrect Choice

D. Iron

E. Hydration
Methylene blue is redued in the presnce of NADPH and diaphorase II to leukomethylene blue, which
then reduces methemoglobin (Fe3+) to hemoglobin (Fe2+)


110) Which biologic agent is infused intravenously?

A. Etanercept

B. Alefacept

C. InfliximabCorrect Choice

D. Efalizumab

E. None of these answers are correct
Infliximab is infused intravenously


111) Which medication is the most likely cause of this drug eruption?

A. Sulfur based medications

B. Nonsteroidal anti-inflammatory medications

C. Methotrexate

D. Beta-lactam antibioticsCorrect Choice

E. Calcium channel blockers
Acute generalized exanthematous pustulosis (AGEP)is characterized by small pustules on
erythematous base with evenutal desqumation. As opposed to most drug reactions, AGEP may
occur within one week, 50% occur within the first 24 of exposure


112) The mechanism action of this cytotoxic agent is via inhibition of IMP dehydrogenase.

A. Mycophenolate mofetilCorrect Choice

B. Azathioprine

C. Hydroxyurea

D. Methotrexate

E. 5-fluorouracil
Mycophenolate mofetil (cellcept), a purine analog, blocks de novo purine synthesis by inhibiting the
enzyme inosine monophosphate dehydrogenase


113) All of the following are reported cutaneous side effects of zidovudine EXCEPT:

A. Trichomegaly

B. Periungual pyogenic granulomas

C. Diffuse and oral hyperpigmented macules

D. None of the above (all are reported side effects)Correct Choice




                                                 29
E. Hyperpigmented streaks in nails
All of the following are cutaneous side effects reported with zidovudine, a nucleoside HIV reverse
transcriptase inhibitor. Periungal/paronychial eruptions resulting in pyogenic granuloma-like lesions
have also been reported with other HIV medications, including indinavir and lamivudine


114) Which of the following biologic therapies is pregnancy category C?

A. Infliximab

B. Alefacept

C. Etanercept

D. EfalizumabCorrect Choice

E. Adalimumab
All of the above drugs are pregnancy category B except efalizumab which is category C


115) Which of the following may cause an acneiform eruption?

A. ACTH Correct Choice

B. Fluoxetine

C. Finasteride

D. Valproic acid

E. Methotrexate
Many medications are associated with acneiform eruptions, including halogens (bromide and
iodide), androgenic hormones such as testosterone, ACTH, corticosteroids, isoniazid (INH), lithium,
phenytoin, and vitamins B2, B6 and B12


116) Which cell type is increased by glucocorticoids?

A. Neutrophils Correct Choice

B. Monocytes

C. T-cells

D. Eosinophils

E. B-cells
Glucocorticoids alter the balance of circulating leukocytes, causing an increase in the number of
polymorphonuclear leukocytes and diminishing the numbers of lymphocytes, eosinophils, and
monocytes


117) Over use of of which medication may lead to this clinical image?

A. Topical antibiotic

B. Topical steroid

C. Calcipotriene

D. Imiquimod

E. HydroquinoneCorrect Choice




                                                 30
Exogenous ochronosis has been reported with prolonged use of high concentration hydroquinone.
On pathology, a characteristic ochre colored deposit is noted between the collagen bundles


118) Which of the following events is most important in the pathogenesis of this painful eruption?

A. Increased expression of FasLCorrect Choice

B. Reduction in circulating IL-6

C. Cleavage of desmoglein 1

D. Reduction in circulating tumor necrosis factor

E. Overexpression of keratins 6 and 16
Toxic epidermal necrolysis is a life threatening drug eruption characterized by widespread epidermal
necrosis. The exact etiology of the keratinocyte necrosis has not been fully elucidated. However,
FasL (FasL and Fas are able to trigger apoptosis) has been shown to be upregulated in TEN


119) Which antiparasitic agent acts by inhibiting fumarate reductase?

A. Cidofovir

B. ThiabendazoleCorrect Choice

C. Ivermectin

D. Permethrin

E. Lindane
Thiabendazole inhibits fumarate reductase, a helminth-specific enzyme. It is used to treat creeping
eruption or cutaneous larva migrans and larva currens. Ivermectin blocks glutamate-gated chloride
ion channels, and is used to treat strongyloidiasis, onchocerciasis, and Norwegian scabies. Lindane
is an organochloride which blocks neural transmission, and is effective against scabies, pubic lice,
head lice, and body lice. Permethrin disables sodium transport channels in the nerve cell membrane
of the parasite. Cidofovir is an antiviral nucleotide analogue


120) Which chemotherapeutic agent has been reported to cause acral sclerosis with Raynaud's
phenomenon?


A. BleomycinCorrect Choice

B. Methotrexate

C. Interferon

D. Actinomycin

E. 5-Fluorouracil
Bleomycin is an antibiotic that induces single strand breaks in the DNA. Reactions to bleomycin
include flagellae hyperpigmentation, acral sclerosis with Reynaud's, penile calcification, and a
morbilliform eruption


121) Which of the following medications is most likely to have caused this reaction?

A. Lithium

B. Diclofenac

C. Vancomycin




                                                    31
D. Pseudoephedrine hydrochlorideCorrect Choice

E. ACE inhibitor
Fixed drug eruptions occur 30 mintues to 8 hours after ingestion of offending agent. After
rechallenge with the same agent, the fixed drug eruption will recur. Potential causes of a fixed drug
eruption include analgesics, sulfonamides, barbituates, pseudoephedrine and anticonvulsants


122) Which of the following chemotherapeutic agents has been linked to acneiform eruptions?

A. CetuximabCorrect Choice

B. Doxorubicin

C. Cytarabine

D. Bleomycin

E. Cisplatin
Cetuximab is a chimeric anti-epidermal growth factor receptor antibody that is FDA approved to
treat advanced colorectal cancer. Acneiform eruptions have been reported to occur in up to 1/3 of
patients


123) What family of medications is associated with xerosis?

A. Sulfonylureas

B. Cholesterol lowering agentsCorrect Choice

C. Beta blockers

D. Loop diuretics

E. Calcium channel blockers
Medications that alter the lipid composition of the epidermis and stratum corneum may impair the
normal barrier function of the skin. Cholesterol lowering medications like HMG-CoA reductase
inhibitors and niacin may cause xerosis through this mechanism


124) Which of the following are retinoid side effects?

A. Reversible hyperthyroidism

B. Delayed epiphyseal closure

C. Diffuse interstitial skeletal hypo-ostosis

D. All of these answers are corect

E. Pseudotumor cerebriCorrect Choice
Bexarotene has been shown to cause reversible hypothyroidism, not hyperthyroidism. Systemic
retinoids have been shown to cause diffuse interstitial skeletal hyperostosis, premature epiphyseal
closure, and pseudotumor cerebri (risk increased with concommitant use of tetracyclines


125) Which of the following is a potentially irreversible ocular side effects of antimalarial agents?

A. Pterygium

B. RetinopathyCorrect Choice

C. Cataracts




                                                  32
D. Neuromuscular eye toxicity

E. Corneal deposition - causing halos, blurred vision, photophobia
Three types of ocular adverse effects may develop from antimalarials: corneal deposits,
neuromuscular eye toxicity and retinopathy. Only retinopathy is potentially irreversible. It is
recommended that a patient be evaluated for retinopathy at baseline, then every 6 months by an
ophthamologist. Testing visual acuity, visual fields and performing a funduscopic examination are
considered acceptable for screening purposes


126) Clinical evidence of hypothyroidism can be induced by which drug?

A. Isotretinoin

B. Gold

C. Bexarotene Correct Choice

D. Griseofulvin

E. Acitretin
Bexarotene can cause reversible elevations in TSH levels and reductions in total T4 levels,
associated with mild symptoms of hypothyroidism


127) Which of the following agents is NOT a UVB blocker?

A. SulisobenzoneCorrect Choice

B. Padimate A

C. PABA

D. Salicylates

E. Cinnamates
Sulisobenzone is a benzophenone, a UVA blocker. The others are UVB blockers. Padimate A is a
PABA derivative


128) Which of the following drugs has been known to cause penile erosions?

A. Penciclovir

B. Abacavir

C. Cidofovir

D. Gancyclovir

E. FoscarnetCorrect Choice
Foscarnet has been reported to cause penile erosions


129) Peak vulnerability to thalidomide occurs between which days of gestation?

A. Days 15-20

B. Days 37-56

C. Days 21-36Correct Choice

D. Days 57-70




                                                 33
E. Days 1-14
Peak vulnerability to thalidomide occurs between days 21 to 36 of gestation, during which only a
single dose will cause birth defects to occur. Birth defects associated with thalidomide include
phocomelia (underdevelopment of arms and legs, the most common birth defect), ear
malformation, and gastrointestinal and urogenital defects


130) Extension of this blister by application of perpendicular pressure is an example of what?

A. Gorlin's sign

B. Hutchinson's sign

C. Nikolsky sign

D. Fitzpatrick sign

E. Asboe-Hansen's signCorrect Choice
Toxic epidermal necrolysis is a serious and potentially life threatening reaction to medications. The
most commonly implicated medications include penicillins, NSAIDS, and anti-convulsants. Asboe-
Hansen's sign results in extension of a blister with perpendicular pressure. Nikolsky sign is the
separation of epidermis from the dermis by application of tangential mechanical pressure


131) Which of the following side effects has not been reported in association with intravenous
immune globulin?


A. Hypotension

B. Headache

C. Stevens-Johnson syndromeCorrect Choice

D. Anaphylaxis

E. Flushing
IVIG is used to treat several diseases including graft versus host disease, connective tissue disease,
and autoimmune bullous dermatoses. Adverse effects include infusion reactions (headache,
flushing, chills, myalgia, wheezing, achycardia, lower back pain, nausea, or hypotension).
Anaphylaxis occurs rarely. Disseminated intravascular coagulation, transient neutropenia, and
aseptic meningitis syndrome has been reported. Cutaneous adverse effects include eczematous
eruptions and alopecia


132) The form of erythromycin most likely to cause jaundice is:

A. EstolateCorrect Choice

B. Gluceptate

C. Lactobionate

D. Ethylsuccinate

E. Stearate
The form of erythromycin that most likely causes jaundice is estolate


133) Which of the following is known to induce lichen planus-like eruptions?

A. Doxepin

B. Minocycline



                                                  34
C. Mercury

D. Gold Correct Choice

E. Dapsone
Mucocutaneous side effects of gold include stomatitis, cheilitis, lichen planus- like eruptions, and
pityriasis rosea-like eruptions


134) Which of the following medications is most likely to result in increased carbamazepine levels?

A. Rifampin

B. Minocycline

C. Erythromycin Correct Choice

D. Azithromycin

E. TMP-SMX
Eythromycin inhibits the hepatic cytochrome P450 system and can increase serum levels and
potential toxicities of carbamazapene, theophylline, warfarin, digoxin, methylprednisolone


135) Which of the following statements regarding drug interactions is true?

A. Tobacco induces P-450 enzymes

B. All of the above are trueCorrect Choice

C. Drugs that induce CYP3A enzymes may decrease levels of drugs which act as substrates for
CYP3A

D. CYP3A inhibitors may increase levels and cause toxicity of drugs metabolized by cytochrome
P-450

E. Terbinafine is not metabolized by cytochrome P-450
The most relevant drug interactions in dermatology involve the hepatic biotransformation pathways
catalyzed by the cytochrome P-450 isoenzymes from the subfamilies CYP3A3/4. Drugs that induce
CYP3A enzymes may decrease levels of drugs which act as substrates for CYP3A. CYP3A inhibitors
may increase levels and cause toxicity of drugs metabolized by cytochrome P-450. Terbinafine is
not metabolized by cytochrome P-450, but by cytochrome 2B6 instead. Tobacco induces P-450
enzymes


136) The anti-viral agent used most often for acyclovir-resistent HSV and VZV infections is:

A. Pencyclovir

B. Valacyclovir

C. Foscarnet Correct Choice

D. Famcyclovir

E. Gancyclovir
Foscarnet does not require phosphorylaton for antiviral activity. Thereofre, it is achieve against
viruses resistant to acyclovir, famcyclovir, or gancyclovir on the basis of altered-kinase activities


137) What tetracycline is not phototoxic?

A. Doxycycline




                                                   35
B. Tetracycline

C. Minocycline Correct Choice

D. Demeclocycline

E. Oxytetracycline
Minocycline is not phototoxic. Demeclocycline and doxycycline are the most phototoxic of all the
tetracyclines. Onycholysis can accompany tetracycline-induced phototoxicity


138) Anemia, leg ulcers, poikilodermatous skin changes, hepatitis, renal toxicity, and acral
erythema are most commonly associated with what medication?


A. 5-fluorouracil

B. Doxorubicin

C. Methotrexate

D. Hydroxyurea Correct Choice

E. Cyclosporine
The constellation of adverse effects is most closely associated with hydroxyurea


139) What drug can potentiate bone marrow suppression when used concomitantly with
azathioprine?


A. Salicylates

B. NSAIDS

C. Sulfonamides

D. Allopurinol Correct Choice

E. Phenytoin
Concomitant allopurinol use, which inhibits xanthine oxidase, can lead to excess toxic purine
analogs via increased metabolism of azaithioprine via the HGPRT pathway, causing bone marrow
suppression


140) What is the mechanism utilized by the co-administration of probenicid to raise blood levels of
penicillins in patients with infections that require high blood levels?


A. Synergistic effect of probenicid with penicillins

B. Displacement of plasma proteins

C. Competitive inhibition of b-lactam binding sites

D. Inhibition of cytochrome P-450 hepatic biotransformation system

E. Prolongs the half-life of penicillins by decreasing renal tubular secretion Correct Choice
Probenicid is co-administered with penicillin to prolong its half-life through decreased renal tubular
secretion when higher blood levels are warranted


141) Which of the following statements regarding the ocular toxicities of the antimalarial drugs is
NOT true?


A. Chloroquine and hydroxychloroquine should not be given together because of an additive effect on




                                                       36
retinotoxicity

B. True retinopathy is associated with "bull's eye" pigment deposition, central scotoma, and
diminished visual acuity

C. Premaculopathy associated with changes in visual fields without visual loss is reversible if the
antimalarial is discontinued

D. The 4-aminoquinolones may have significant associated ocular toxicity

E. Risk of retinopathy is greatest for quinacrine, followed by chloroquineCorrect Choice
Risk of retinopathy is greatest with chloroquine and does not exist for quinacrine


142) Which of the following statements regarding podophyllin is NOT correct?

A. It arrests cells in telophaseCorrect Choice

B. It is derived from the May apple plant

C. It binds tubulin

D. None (all of these statements are true)

E. It is contraindicated in pregnancy
Podophyllin is a crude cytotoxic extract from the May apple plant. It is antimitotic, arresting cells in
metaphase (not telophase) by binding to the protein tubulin. It may be teratogenic and should not
be used in pregnancy


143) Which cutaneous side effect is a common complication of nitrogen mustard therapy?

A. Telangiectasia

B. Allergic contact dermatitisCorrect Choice

C. Hyperpigmentation

D. Bullous drug eruption

E. Fixed drug eruption
Topical nitrogen mustard, or mechlorethamine, is an antineoplastic agent which works via alkylation
thereby inhibiting DNA synthesis. Allergic contact dermatitis occurs in two-thirds of patients who are
treated with topical nitrogen mustard in aqueous solution, but occurs in less than 5% of patients
treated with the ointment based preparation


144) Cefaclor has been associated with increased incidence of what in children?

A. Transaminitis

B. Anaphylaxis

C. Generalized tonic-clonic seizures

D. Mononucleosis-like syndrome

E. Serum sickness reaction Correct Choice
The use of cefaclor has been associated with an increased incidence of serum sickness in children


145) Which of the following oral agents has been effective in the treatment of Norwegian scabies?

A. Metroniddazole



                                                   37
B. IvermectinCorrect Choice

C. Griseofulvin

D. Mebendazole

E. Thiabendazole
Ivermectin (Stromectol) is an anti-helminthic agent currently FDA-approved for the treatment of
strongyloides and onchocerciasis. Several publications have reported efficacy of this agent in the
treatment of scabies and head lice. Due to its low rate of adverse effects, its high rate of
effectiveness, and its ease of administration, some authors consider this agent to be the treatment
of choice for scabies and head lice. The mechanism of action of ivermectin is blockade of glutamate-
gated, chloride ion channels, with adverse effects on nerve and muscle resulting in paralysis and
death of the helminth or mite. The drug has a very low affinity for mammalian chloride channels
resulting in its relatively low toxicity. Adverse effects are rare and have been associated with
accidental intoxication. It should be avoided when there is compromise of the blood-brain-barrier.
Ivermectin is pregnancy category C. There are virtually no associated drug interactions with oral
ivermectin therapy


146) Of the medications listed below, the safest to use during pregnancy is:

A. NSAIDS

B. Estrogens

C. Erythromycin estolate

D. Doxycycline

E. Penicillin Correct Choice
Erythromycin estolate is contraindicated in pregnancy because of the risk of cholestatic hepatitis.
Other forms of erythromycin are safer for use in pregnancy. Tetracyclines are category D and
estrogens are category X. NSAIDS may promote persistent fetal circulation or oligohydramnios


147) Which of the following antifungal agents is contraindicated in patients with a history of
porphyria?


A. Ketoconazole

B. Terfinafine

C. Itraconazole

D. GriseofulvinCorrect Choice

E. Fluconazole
Griseofulvin has been reported as a potential exacerbator of acute intermittent porphyria, and thus
is contraindicated in patients with a history of porphyria


148) Which of the following statements regarding dapsone and sulfapyridine is true?

A. Sulfapyridine has a similar but often more severe side effect profile than dapsone

B. Dapsone hypersenstivity syndrome is characterized by lymphocytosis

C. They exert their anti-inflammatory actions by stimulating the myeloperoxidase activity of
polymorphonuclear leukocytes

D. None of these answers are correct (all are false)Correct Choice

E. Concomittant administration of cimetidine has been shown to increase the risk of




                                                 38
methemoglobinemia
None of the above statements are true. Dapsone and sulfapyridine exert their anti-inflammatory
actions by inhibiting the myeloperoxidase activity and chemotactic abilities of polymorphonuclear
leukocytes. Dapsone hypersenstivity syndrome is characterized by eosinophilia, as well as a severe
mononucleosis-like reaction, including fever, erythroderma, hepatitis, and even death. Sulfapyridine
has a similar but often less severe side effect profile. Cimetidine has been shown to provide some
protection against methemoglobin formation


149) Which of the following statements is true regarding the absorption of antibiotics?

A. Tetracycline absorption is impaired by the ingestion of calcium products but not iron

B. Doxycycline absorption is impaired by the ingestion of dairy products and calcium

C. None of the above (all of the above statements are false)Correct Choice

D. Minocycline absorption is not significantly impaired by the ingestion of calcium products but it
should be taken on an empty stomach

E. Fluoroquinolone absorption is not altered by antacids
Antacids decrease the absorption of fluoroquinolones and should be taken at least 2 hours after the
drug. Tetracycline absorption is impaired by the ingestion of dairy products, calcium, and iron or
zinc salts. Minocycline and doxycycline absorption is not impaired by the ingestion of those
products; they may be taken on an empty stomach or with food. Thus, all of the statements are
false


150) Treatment with isotretinoin has been shown to cause increased colonization of the skin with
which of the following organisms?


A. Pseudomonas aeruginosa

B. Staphylococcus aureusCorrect Choice

C. Demodex folliculorum

D. Pityrosporum orbiculare

E. Streptococcus pyogenes
Staphylococcus aureus colonization tends to correlate with isotretinoin-induced reduction in sebum
production and may lead to infections. This complication may possibly be prevented with pulsed
intranasal mupirocen therapy. There has been a report of staphylococcus endocarditis in a patient
with underlying aortic insufficiency


151) This retinoid targets RXR receptors:

A. Etretinate

B. Isotretinoin

C. Tretinoin

D. Acitretin

E. BexaroteneCorrect Choice
Bexarotene targets RXR receptors. It is used in the treatment of mycosis fungoides refractory to
conventional therapy. Side effects include central hypothyroidism and hyperlipidemia. These should
be treated with synthroid, lipitor, and fenofibrate


152) Neutrophilic eccrine hidradenitis is a side effect of which therapeutic agent?


                                                  39
A. Interferon-alpha

B. Granulocyte colony stimulating factor

C. Cytarabine Correct Choice

D. Bieomycin

E. Intravenous immune globulin
Neutrophilic eccrine hidradenitis most commonly occurs in the setting of a patient with acute
myelogenous leukemia being treated with cytarabine. Clinical manifestations include tender,
erythematous macules, papuls and plaques on the trunk, neck and extremities which resolve within
a few days. Histologically, this drug eruption is defined by the presence of dense neutrophilic
infiltrate within and around eccrine glands, with necrosis of eccrine epithelial cells


153) Gray-green discoloration of the mid-portion of permanent teeth is a side effect of?

A. Doxycycline

B. Fluoroquinolones

C. Minocycline Correct Choice

D. Tetracycline

E. clindamycin
In contrast to tetracycline staining of the teeth, which occurs in childhood and produces a brown
discoloration along the gingival third, minocycline stains the permanent teeth in adults, with a gray-
green discoloration of the mid-portion of the tooth


154) Side effects of PUVA include all of the following except:

A. Headache

B. Nausea

C. Pruritus

D. NeutropeniaCorrect Choice

E. Insomnia
Side effects of psoralen with ultraviolet A light (PUVA) include side effects which are due to
phototoxic effects including pruritus, photoonycholysis, friction blisters, ankle edema and
hypertrichosis. In addition, there are adverse effects from methoxypsoralen including
gastrointestinal and neurologic effects, hepatotoxicity and exanthems. Neutropenia is not a
described side effect of PUVA


155) A patient taking daily prednisone is advised to switch to alternate day dosing to decrease the
risk of:


A. Adrenal crisisCorrect Choice

B. Glaucoma

C. Aseptic bone necrosis

D. Cataracts

E. Osteoporosis
Long-term therapy with oral corticosteroids can result in numerous adverse effects, including
elevated risks of glaucoma, cataracts, hypertension, diabetes, osteoporosis, adrenal axis




                                                  40
suppression, and aseptic bone necrosis. Alternate-day dosing or oral corticosteroids lowers the rate
of adrenal axis suppression. It is hypothesized that during the off day, cell mediated immunity,
white blood cells subset levels, and potassium excretion are normalized while the anti-inflammatory
benefits of the drug persist. Alternate-day corticosteroid therapy should be employed once adequate
disease control has been attained with daily dosing. Of note, the risk of cataracts, osteoporosis, and
other adverse effects of long-term corticosteroid use are not minimized with alternate-day dosing


156) Penile erosions are a reported side effect associated with which medication?

A. Trimethoprim-sulfamethoxazole

B. Azaithioprine

C. Bleomycin

D. Acyclovir

E. Foscarnet Correct Choice
None


157) Which of the following biologic agents is pregnancy category C?

A. All of these answers are correct

B. Etanercept

C. Alefacept

D. Infliximab

E. EfalizumabCorrect Choice
Efalizumab is pregnancy category C. The other drugs listed are pregnancy category B


158) Which of the following is NOT known to increase methotrexate levels?

A. None of these answers are correct(all are known to increase methotrexate levels)Correct Choice

B. Tetracyclines

C. Salicylates

D. NSAIDs

E. Phenothiazines
Tetracyclines, phenytoin, phenothiazines, chloramphenicol, NSAIDs, salicylates, and sulfonamides,
among other drugs, can all increase methotrexate levels by displacement of plasma proteins


159) What is the best medication to lower isotretinoin induced hypertriglyceridemia?

A. Cholestyramine

B. GemfibrozilCorrect Choice

C. All are equally effective

D. Simvastatin

E. Niacin
Gemfibrozil generally reduces trygliceride levels to a greater extent than niacin, cholestyramine,
and the HMG-CoA reductase inhibitors




                                                 41
160) Keratinocyte differentiation is enhanced by retinoids with all of the following EXCEPT:

A. Increased keratohyalin granules

B. Increased filaggrin production

C. Stimulation of ornithine decarboxylaseCorrect Choice

D. Odland body secretion of lipids

E. Increased keratin filaments
Keratinocyte differentiation is enhanced by retinoids with increased filaggrin production, increased
keratohyalin granules, keratin filaments, and Odland body secretion of lipids. Retinoids directly
inhibit ornithine decarboxylase and therefore lessen inflammatory hyperplasia


161) Which member of the tetracycline family is most likely to have caused this photomediated
reaction?


A. Doxycycline

B. Oxytetracycline

C. Minocycline

D. DemeclocyclineCorrect Choice

E. Tetracycline
The tetracycline family of antibiotics are bacteriostatic and act by inhibiting protein synthesis. Each
member of the family may cause photosensitivity, but demeclocycline is the most photosensitizing


162) The treatment of choice for erythema nodosum leprosum (ENL) is:

A. Rifampin and clofazamine

B. Clofazamine

C. Rifampin

D. Thalidomide Correct Choice

E. Isoniazid, rifampin and clofazamine
NEEDS EXPLANATIONS


163) The laboratory abnormality most associated with cyclosporine is:

A. Hyperkalemia Correct Choice

B. Hypermagnesemia

C. Hypouricemia

D. Increased LDH

E. Hyponatremia
The laboratory abnormalities associated with cyclosporine are decreased magnesium, increased
potassium, and increased uric acid. Renal function and blood pressure must also be monitored
closely in patients using cyclosporine




                                                  42
164) Concomitant use of methotrexate and what other drug is contraindicated because of the
potential increased risk of pancytopenia?


A. Tetracycline

B. NSAIDs Correct Choice

C. Systemic retinoids

D. Folic acid

E. Acetaminophen
Drugs that simultaneously inhibit the folate metabolic pathway, such as NSAIDS, dapsone, or
trimethoprim-sulfamethoxazole, can increase hematologic toxicity when combined with
methotrexate


165) What is the treatment of choice for CMV retinitis?

A. Ganciclovir

B. Combovir

C. FoscarnetCorrect Choice

D. Cidofovir

E. Acyclovir
Foscarnet is indicated for the treatment of CMV retinitis and acyclovir resistant HSV, both of which
can been seen in HIV patients. Foscarnet acts by inhibiting viral specific DNA polymerase. Unlike
acyclovir, it does not require activation by thymidine kinase. Nephrotoxicity and seizures are major
side effects


166) What is the most common cause of nonpigmented fixed drug eruption?

A. Tetracycline

B. Sulfonamides

C. Naproxen

D. Phenolphthalein

E. PseudoephedrineCorrect Choice
Pigmented incontinence is usually prominent in a fixed drug eruption; yet occasionally, fixed drug
reactions do not result in long-lasting hyperpigmentation. The so-called nonpigmented fixed drug
eruption is distictive. Pseudoephedrine hydrochloride is by far the most common cause


167) Which drug is the most likely to cause these changes in the gingiva?

A. Doxycycline

B. Acitretin

C. Methotrexate

D. CyclosporineCorrect Choice

E. Cephalexin
Gingival hyperplasia is a well described reaction to multiple medications including cyclosporine,
phenytoin, and calcium channel blockers. All of these medications affect the influx of calcium into
the cells which enhances the function of fibroblasts



                                                 43
168) The anti-CD 20 antibody rituximab is FDA-approved for treatment of which of the following?

A. Paraneoplastic pemphigus

B. Mycosis fungoids

C. Non-Hodgkin’s lymphomaCorrect Choice

D. Psoriasis

E. Metastatic melanoma
Rituximab (brand name Rituxan) is a monoclonal antibody is approved for the treatment of CD20
non-Hodgkin’s lymphoma. Rituximab is a monoclonal antibody directed against B lymphocytes
which are CD20.


169) The combination of doxorubicin and which medications has been reported to cause sticky
skin?


A. Amphotercin

B. G-CSF

C. Vancomycin

D. Cisplatin

E. KetoconazoleCorrect Choice
Polsen et. al. reported a 29% incidence of sticky skin in patients treated with high dose
ketoconazole and doxorubicin for prostate cancer. Other medications reported to cause this include
etretinate and tretinoin.


Polsen JA, Cohen PR, Sella: Acquired cutaneous adherence in patients with androgen-independent
prostate cancer receiving ketoconazole and doxorubicin: medication-induced sticky skin. J Am Acad
Dermatol: 32 (4):571-5 1995


170) Which of the following cytotoxic agents has been associated with poikiloderma of the dorsal
hands with a band-like distribution of the fingers and toes?


A. Intralesional bleomycin

B. HydroxyureaCorrect Choice

C. Azathioprine

D. Doxorubicin

E. Flurouracil
Hydroxyurea has been associated with poikiloderma of the dorsal hands with a band-like distribution
of the fingers and toes. It has also been associated with diffuse hyperpigmentation, and with leg
ulcers upon withdrawal


171) If a patient develops a dilantin hypersensitivity reaction, which anticonvulsant is the better
alternative therapy?


A. Phenobarbital

B. None of these answers are correct




                                                  44
C. Valproic acid Correct Choice

D. Phenytoin

E. Carbamezapine
Carbamezapine, phenytoin, and phenobarbital are known to cross-react with one another


172) Hemorrhagic cystitis is a risk of what chemotherapy?

A. Chlorambucil

B. Hydroxyurea

C. Vinca alkaloids

D. Cyclophosphamide Correct Choice

E. Azaithioprine
Bladder toxicity is due to the acrolein metabolite of cyclophosphamide. Hemorrhagic cystitis is
associated with the increased risk of transitional cell carcinoma of the bladder. Mesna has been used
to reduce the toxic effect. The risk of cystitis is avoided by adequate fluid intake, frequent voiding,
and careful screening for hematuria


173) All of the following are true regarding water-soluble retinoids EXCEPT:

A. They include bexarotene

B. They include etretinateCorrect Choice

C. They include isotretinoin

D. They are undetectable in the serum after 1 month of stopping therapy

E. They have very little lipid deposition
Isotretinoin, acitretin, and bexarotene are water-soluble, with very little lipid deposition. Etretinate
is 50 times more lipophilic than acitretin, with increased storage in adipose tissue

The anti-HIV medication best known for causing a severe reaction which can result in fatality upon
rechallenge is:


A. Zidovudine

B. Indinavir

C. Nevirapine

D. Abacavir Correct Choice

E. Didanosine
The hypersensivity reaction associated with abacavir usually resolves with cessation of the drug,
however upon rechallenge the reaction can be life-threatening




                                                   45

More Related Content

What's hot

ETAS_MCQ_03 a genodermatoses
ETAS_MCQ_03 a genodermatosesETAS_MCQ_03 a genodermatoses
ETAS_MCQ_03 a genodermatosesDerma202
 
ETAS_MCQ_15 dermatologic and cosmetic surgery
ETAS_MCQ_15 dermatologic and cosmetic surgeryETAS_MCQ_15 dermatologic and cosmetic surgery
ETAS_MCQ_15 dermatologic and cosmetic surgeryDerma202
 
MCQ August with answers - Dr Ameen Alawadhi
MCQ August with answers - Dr Ameen AlawadhiMCQ August with answers - Dr Ameen Alawadhi
MCQ August with answers - Dr Ameen Alawadhiaskadermatologist
 
Arab board primary exam in dermatology 2012
Arab board primary exam  in dermatology 2012Arab board primary exam  in dermatology 2012
Arab board primary exam in dermatology 2012Derma202
 
MCQ July with answers - Dr Ameen Alawadhi
MCQ July with answers - Dr Ameen AlawadhiMCQ July with answers - Dr Ameen Alawadhi
MCQ July with answers - Dr Ameen Alawadhiaskadermatologist
 
Dermatology board review
Dermatology board reviewDermatology board review
Dermatology board reviewAhmed Amer
 
Dermatolody quizzes
Dermatolody quizzesDermatolody quizzes
Dermatolody quizzesFayzaRayes
 
Dermatology MCQ and AAFP.pptx
Dermatology MCQ and AAFP.pptxDermatology MCQ and AAFP.pptx
Dermatology MCQ and AAFP.pptxAbdulaziz Bagasi
 
Skin manifestation of systemic disease
Skin manifestation of systemic diseaseSkin manifestation of systemic disease
Skin manifestation of systemic diseaseahmedalgzali1
 
Biologicals in Deramtology (Part 1 )
Biologicals in Deramtology (Part 1 )Biologicals in Deramtology (Part 1 )
Biologicals in Deramtology (Part 1 )Smruti Ramawanshi
 
Cutaneous mucinoses
Cutaneous mucinosesCutaneous mucinoses
Cutaneous mucinosesheera sanju
 
dermatology.Disorders of skin color.(dr.ali)
dermatology.Disorders of skin color.(dr.ali)dermatology.Disorders of skin color.(dr.ali)
dermatology.Disorders of skin color.(dr.ali)student
 
dermatology.Bact .inf 5th.(dr.ali)
dermatology.Bact .inf 5th.(dr.ali)dermatology.Bact .inf 5th.(dr.ali)
dermatology.Bact .inf 5th.(dr.ali)student
 
Discovery and Development of Methotrexate on Psoriasis
Discovery and Development of Methotrexate on PsoriasisDiscovery and Development of Methotrexate on Psoriasis
Discovery and Development of Methotrexate on PsoriasisGenevieve Tang
 

What's hot (20)

ETAS_MCQ_03 a genodermatoses
ETAS_MCQ_03 a genodermatosesETAS_MCQ_03 a genodermatoses
ETAS_MCQ_03 a genodermatoses
 
ETAS_MCQ_15 dermatologic and cosmetic surgery
ETAS_MCQ_15 dermatologic and cosmetic surgeryETAS_MCQ_15 dermatologic and cosmetic surgery
ETAS_MCQ_15 dermatologic and cosmetic surgery
 
MCQ August with answers - Dr Ameen Alawadhi
MCQ August with answers - Dr Ameen AlawadhiMCQ August with answers - Dr Ameen Alawadhi
MCQ August with answers - Dr Ameen Alawadhi
 
Arab board primary exam in dermatology 2012
Arab board primary exam  in dermatology 2012Arab board primary exam  in dermatology 2012
Arab board primary exam in dermatology 2012
 
MCQ July with answers - Dr Ameen Alawadhi
MCQ July with answers - Dr Ameen AlawadhiMCQ July with answers - Dr Ameen Alawadhi
MCQ July with answers - Dr Ameen Alawadhi
 
Dermatology board review
Dermatology board reviewDermatology board review
Dermatology board review
 
Mrcp 2 dermatology
Mrcp 2 dermatologyMrcp 2 dermatology
Mrcp 2 dermatology
 
Vitiligo
VitiligoVitiligo
Vitiligo
 
Dermatology 5th year, 3rd lecture (Dr. Faraedon Kaftan)
Dermatology 5th year, 3rd lecture (Dr. Faraedon Kaftan)Dermatology 5th year, 3rd lecture (Dr. Faraedon Kaftan)
Dermatology 5th year, 3rd lecture (Dr. Faraedon Kaftan)
 
Dermatolody quizzes
Dermatolody quizzesDermatolody quizzes
Dermatolody quizzes
 
Dermatology MCQ and AAFP.pptx
Dermatology MCQ and AAFP.pptxDermatology MCQ and AAFP.pptx
Dermatology MCQ and AAFP.pptx
 
Mucinoses
MucinosesMucinoses
Mucinoses
 
Skin manifestation of systemic disease
Skin manifestation of systemic diseaseSkin manifestation of systemic disease
Skin manifestation of systemic disease
 
Atopic dermatitis
Atopic dermatitisAtopic dermatitis
Atopic dermatitis
 
Biologicals in Deramtology (Part 1 )
Biologicals in Deramtology (Part 1 )Biologicals in Deramtology (Part 1 )
Biologicals in Deramtology (Part 1 )
 
Cutaneous mucinoses
Cutaneous mucinosesCutaneous mucinoses
Cutaneous mucinoses
 
dermatology.Disorders of skin color.(dr.ali)
dermatology.Disorders of skin color.(dr.ali)dermatology.Disorders of skin color.(dr.ali)
dermatology.Disorders of skin color.(dr.ali)
 
dermatology.Bact .inf 5th.(dr.ali)
dermatology.Bact .inf 5th.(dr.ali)dermatology.Bact .inf 5th.(dr.ali)
dermatology.Bact .inf 5th.(dr.ali)
 
Dermatology 5th year, 1st lecture (Dr. Darseem)
Dermatology 5th year, 1st lecture (Dr. Darseem)Dermatology 5th year, 1st lecture (Dr. Darseem)
Dermatology 5th year, 1st lecture (Dr. Darseem)
 
Discovery and Development of Methotrexate on Psoriasis
Discovery and Development of Methotrexate on PsoriasisDiscovery and Development of Methotrexate on Psoriasis
Discovery and Development of Methotrexate on Psoriasis
 

Viewers also liked

ETAS_MCQ_05 dermatopathology
ETAS_MCQ_05 dermatopathologyETAS_MCQ_05 dermatopathology
ETAS_MCQ_05 dermatopathologyDerma202
 
dermatology.1 eryth, telan, urt & hshp(dr.faraydwn)
dermatology.1 eryth, telan, urt & hshp(dr.faraydwn)dermatology.1 eryth, telan, urt & hshp(dr.faraydwn)
dermatology.1 eryth, telan, urt & hshp(dr.faraydwn)student
 
Dermatology osce slides
Dermatology osce slidesDermatology osce slides
Dermatology osce slidesHabrol Afzam
 
Cutaneous Manifestations of GI Malignancies
Cutaneous Manifestations of GI MalignanciesCutaneous Manifestations of GI Malignancies
Cutaneous Manifestations of GI MalignanciesMohammed Ezz El-din
 
Gastrointestinal mcq
Gastrointestinal mcqGastrointestinal mcq
Gastrointestinal mcqRashed Hassen
 
Skin Manifestations Of Systemic Diseases
Skin Manifestations Of Systemic DiseasesSkin Manifestations Of Systemic Diseases
Skin Manifestations Of Systemic Diseasesguestfb96c70
 

Viewers also liked (10)

ETAS_MCQ_05 dermatopathology
ETAS_MCQ_05 dermatopathologyETAS_MCQ_05 dermatopathology
ETAS_MCQ_05 dermatopathology
 
Dermatology made easy
Dermatology made easyDermatology made easy
Dermatology made easy
 
dermatology.1 eryth, telan, urt & hshp(dr.faraydwn)
dermatology.1 eryth, telan, urt & hshp(dr.faraydwn)dermatology.1 eryth, telan, urt & hshp(dr.faraydwn)
dermatology.1 eryth, telan, urt & hshp(dr.faraydwn)
 
Dermatology osce slides
Dermatology osce slidesDermatology osce slides
Dermatology osce slides
 
Fe
FeFe
Fe
 
Serofuloderma
SerofulodermaSerofuloderma
Serofuloderma
 
Cutaneous Manifestations of GI Malignancies
Cutaneous Manifestations of GI MalignanciesCutaneous Manifestations of GI Malignancies
Cutaneous Manifestations of GI Malignancies
 
Gastrointestinal mcq
Gastrointestinal mcqGastrointestinal mcq
Gastrointestinal mcq
 
HCV Story ---by Mohammed Hussien
HCV Story ---by Mohammed HussienHCV Story ---by Mohammed Hussien
HCV Story ---by Mohammed Hussien
 
Skin Manifestations Of Systemic Diseases
Skin Manifestations Of Systemic DiseasesSkin Manifestations Of Systemic Diseases
Skin Manifestations Of Systemic Diseases
 

Similar to ETAS_MCQ_16 dermatological drugs

Antiviral agents
Antiviral agentsAntiviral agents
Antiviral agentsHamseHaybe
 
Antiviral agents
Antiviral agentsAntiviral agents
Antiviral agentsHamseHaybe
 
Antiviral agents
Antiviral agentsAntiviral agents
Antiviral agentsHamseHaybe
 
Antifungal Agents Final.pptx
Antifungal Agents Final.pptxAntifungal Agents Final.pptx
Antifungal Agents Final.pptxvaishnavvr360
 
Antifungal Agents.pptx
Antifungal Agents.pptxAntifungal Agents.pptx
Antifungal Agents.pptxHagerAttiya1
 
NEET PG Recall- PART I.pptx
NEET PG Recall- PART I.pptxNEET PG Recall- PART I.pptx
NEET PG Recall- PART I.pptxShivankan Kakkar
 
Antifungal agents
Antifungal agentsAntifungal agents
Antifungal agentsusabwimana
 
PYQ FOR ORR.pptx
PYQ FOR ORR.pptxPYQ FOR ORR.pptx
PYQ FOR ORR.pptxvishvata
 
Antimicrobial
AntimicrobialAntimicrobial
AntimicrobialSanjogBam
 
Block G pre proff KGMC 2022 print with key.pdf
Block G pre proff KGMC 2022 print with key.pdfBlock G pre proff KGMC 2022 print with key.pdf
Block G pre proff KGMC 2022 print with key.pdfKhan676722
 
Antifungal agents
Antifungal agentsAntifungal agents
Antifungal agentsraj kumar
 
Antifungal agents
Antifungal agentsAntifungal agents
Antifungal agentsraj kumar
 
Shanmukh ppt omr
Shanmukh ppt omrShanmukh ppt omr
Shanmukh ppt omrshannu511
 
Anti-Fungal drugs
Anti-Fungal drugsAnti-Fungal drugs
Anti-Fungal drugsEneutron
 
CHEMOTHERAPY_RDP_GENERAL PRINCIPLES OF CHEMOTHERAPY.pdf
CHEMOTHERAPY_RDP_GENERAL PRINCIPLES OF CHEMOTHERAPY.pdfCHEMOTHERAPY_RDP_GENERAL PRINCIPLES OF CHEMOTHERAPY.pdf
CHEMOTHERAPY_RDP_GENERAL PRINCIPLES OF CHEMOTHERAPY.pdfrishi2789
 
Drugs in Haematological Disorders
Drugs in Haematological DisordersDrugs in Haematological Disorders
Drugs in Haematological DisordersMirza Anwar Baig
 

Similar to ETAS_MCQ_16 dermatological drugs (20)

Antiviral agents
Antiviral agentsAntiviral agents
Antiviral agents
 
Antiviral agents
Antiviral agentsAntiviral agents
Antiviral agents
 
Antiviral agents
Antiviral agentsAntiviral agents
Antiviral agents
 
Antifungal Agents Final.pptx
Antifungal Agents Final.pptxAntifungal Agents Final.pptx
Antifungal Agents Final.pptx
 
Anti fungal
Anti fungalAnti fungal
Anti fungal
 
Antifungal Agents.pptx
Antifungal Agents.pptxAntifungal Agents.pptx
Antifungal Agents.pptx
 
NEET PG Recall- PART I.pptx
NEET PG Recall- PART I.pptxNEET PG Recall- PART I.pptx
NEET PG Recall- PART I.pptx
 
Antifungal agents
Antifungal agentsAntifungal agents
Antifungal agents
 
Anti microbial drugs
Anti microbial drugsAnti microbial drugs
Anti microbial drugs
 
PYQ FOR ORR.pptx
PYQ FOR ORR.pptxPYQ FOR ORR.pptx
PYQ FOR ORR.pptx
 
Antimicrobial
AntimicrobialAntimicrobial
Antimicrobial
 
Block G pre proff KGMC 2022 print with key.pdf
Block G pre proff KGMC 2022 print with key.pdfBlock G pre proff KGMC 2022 print with key.pdf
Block G pre proff KGMC 2022 print with key.pdf
 
Antifungal drugs
Antifungal drugs Antifungal drugs
Antifungal drugs
 
Antifungal agents
Antifungal agentsAntifungal agents
Antifungal agents
 
Antifungal agents
Antifungal agentsAntifungal agents
Antifungal agents
 
Shanmukh ppt omr
Shanmukh ppt omrShanmukh ppt omr
Shanmukh ppt omr
 
Antifungal Drugs 3.ppt
Antifungal Drugs 3.pptAntifungal Drugs 3.ppt
Antifungal Drugs 3.ppt
 
Anti-Fungal drugs
Anti-Fungal drugsAnti-Fungal drugs
Anti-Fungal drugs
 
CHEMOTHERAPY_RDP_GENERAL PRINCIPLES OF CHEMOTHERAPY.pdf
CHEMOTHERAPY_RDP_GENERAL PRINCIPLES OF CHEMOTHERAPY.pdfCHEMOTHERAPY_RDP_GENERAL PRINCIPLES OF CHEMOTHERAPY.pdf
CHEMOTHERAPY_RDP_GENERAL PRINCIPLES OF CHEMOTHERAPY.pdf
 
Drugs in Haematological Disorders
Drugs in Haematological DisordersDrugs in Haematological Disorders
Drugs in Haematological Disorders
 

More from Derma202

Phototherapy treatment protocol
Phototherapy treatment protocolPhototherapy treatment protocol
Phototherapy treatment protocolDerma202
 
Histopathplogical photos
Histopathplogical photosHistopathplogical photos
Histopathplogical photosDerma202
 
Slide study from ETAS
Slide  study from ETASSlide  study from ETAS
Slide study from ETASDerma202
 
Dermatology
DermatologyDermatology
DermatologyDerma202
 
ETAS_MCQ_14 plants and creatures of dermatologic significance
ETAS_MCQ_14 plants and creatures of dermatologic significanceETAS_MCQ_14 plants and creatures of dermatologic significance
ETAS_MCQ_14 plants and creatures of dermatologic significanceDerma202
 
ETAS_MCQ_13 photobiology and photosensitivity disorders
ETAS_MCQ_13 photobiology and photosensitivity disordersETAS_MCQ_13 photobiology and photosensitivity disorders
ETAS_MCQ_13 photobiology and photosensitivity disordersDerma202
 
ETAS_MCQ_11 disorder of hair and nails
ETAS_MCQ_11 disorder of hair and nailsETAS_MCQ_11 disorder of hair and nails
ETAS_MCQ_11 disorder of hair and nailsDerma202
 
ETAS_MCQ_03 b genodermatoses
ETAS_MCQ_03 b genodermatosesETAS_MCQ_03 b genodermatoses
ETAS_MCQ_03 b genodermatosesDerma202
 
Derm handbook for medical students and junior doctors 2010
Derm handbook for medical students and junior doctors 2010Derm handbook for medical students and junior doctors 2010
Derm handbook for medical students and junior doctors 2010Derma202
 

More from Derma202 (9)

Phototherapy treatment protocol
Phototherapy treatment protocolPhototherapy treatment protocol
Phototherapy treatment protocol
 
Histopathplogical photos
Histopathplogical photosHistopathplogical photos
Histopathplogical photos
 
Slide study from ETAS
Slide  study from ETASSlide  study from ETAS
Slide study from ETAS
 
Dermatology
DermatologyDermatology
Dermatology
 
ETAS_MCQ_14 plants and creatures of dermatologic significance
ETAS_MCQ_14 plants and creatures of dermatologic significanceETAS_MCQ_14 plants and creatures of dermatologic significance
ETAS_MCQ_14 plants and creatures of dermatologic significance
 
ETAS_MCQ_13 photobiology and photosensitivity disorders
ETAS_MCQ_13 photobiology and photosensitivity disordersETAS_MCQ_13 photobiology and photosensitivity disorders
ETAS_MCQ_13 photobiology and photosensitivity disorders
 
ETAS_MCQ_11 disorder of hair and nails
ETAS_MCQ_11 disorder of hair and nailsETAS_MCQ_11 disorder of hair and nails
ETAS_MCQ_11 disorder of hair and nails
 
ETAS_MCQ_03 b genodermatoses
ETAS_MCQ_03 b genodermatosesETAS_MCQ_03 b genodermatoses
ETAS_MCQ_03 b genodermatoses
 
Derm handbook for medical students and junior doctors 2010
Derm handbook for medical students and junior doctors 2010Derm handbook for medical students and junior doctors 2010
Derm handbook for medical students and junior doctors 2010
 

Recently uploaded

Artifacts in Nuclear Medicine with Identifying and resolving artifacts.
Artifacts in Nuclear Medicine with Identifying and resolving artifacts.Artifacts in Nuclear Medicine with Identifying and resolving artifacts.
Artifacts in Nuclear Medicine with Identifying and resolving artifacts.MiadAlsulami
 
Call Girls Kochi Just Call 9907093804 Top Class Call Girl Service Available
Call Girls Kochi Just Call 9907093804 Top Class Call Girl Service AvailableCall Girls Kochi Just Call 9907093804 Top Class Call Girl Service Available
Call Girls Kochi Just Call 9907093804 Top Class Call Girl Service AvailableDipal Arora
 
Vip Call Girls Anna Salai Chennai 👉 8250192130 ❣️💯 Top Class Girls Available
Vip Call Girls Anna Salai Chennai 👉 8250192130 ❣️💯 Top Class Girls AvailableVip Call Girls Anna Salai Chennai 👉 8250192130 ❣️💯 Top Class Girls Available
Vip Call Girls Anna Salai Chennai 👉 8250192130 ❣️💯 Top Class Girls AvailableNehru place Escorts
 
(👑VVIP ISHAAN ) Russian Call Girls Service Navi Mumbai🖕9920874524🖕Independent...
(👑VVIP ISHAAN ) Russian Call Girls Service Navi Mumbai🖕9920874524🖕Independent...(👑VVIP ISHAAN ) Russian Call Girls Service Navi Mumbai🖕9920874524🖕Independent...
(👑VVIP ISHAAN ) Russian Call Girls Service Navi Mumbai🖕9920874524🖕Independent...Taniya Sharma
 
Call Girl Coimbatore Prisha☎️ 8250192130 Independent Escort Service Coimbatore
Call Girl Coimbatore Prisha☎️  8250192130 Independent Escort Service CoimbatoreCall Girl Coimbatore Prisha☎️  8250192130 Independent Escort Service Coimbatore
Call Girl Coimbatore Prisha☎️ 8250192130 Independent Escort Service Coimbatorenarwatsonia7
 
♛VVIP Hyderabad Call Girls Chintalkunta🖕7001035870🖕Riya Kappor Top Call Girl ...
♛VVIP Hyderabad Call Girls Chintalkunta🖕7001035870🖕Riya Kappor Top Call Girl ...♛VVIP Hyderabad Call Girls Chintalkunta🖕7001035870🖕Riya Kappor Top Call Girl ...
♛VVIP Hyderabad Call Girls Chintalkunta🖕7001035870🖕Riya Kappor Top Call Girl ...astropune
 
Call Girls Horamavu WhatsApp Number 7001035870 Meeting With Bangalore Escorts
Call Girls Horamavu WhatsApp Number 7001035870 Meeting With Bangalore EscortsCall Girls Horamavu WhatsApp Number 7001035870 Meeting With Bangalore Escorts
Call Girls Horamavu WhatsApp Number 7001035870 Meeting With Bangalore Escortsvidya singh
 
Chandrapur Call girls 8617370543 Provides all area service COD available
Chandrapur Call girls 8617370543 Provides all area service COD availableChandrapur Call girls 8617370543 Provides all area service COD available
Chandrapur Call girls 8617370543 Provides all area service COD availableDipal Arora
 
(Rocky) Jaipur Call Girl - 9521753030 Escorts Service 50% Off with Cash ON De...
(Rocky) Jaipur Call Girl - 9521753030 Escorts Service 50% Off with Cash ON De...(Rocky) Jaipur Call Girl - 9521753030 Escorts Service 50% Off with Cash ON De...
(Rocky) Jaipur Call Girl - 9521753030 Escorts Service 50% Off with Cash ON De...indiancallgirl4rent
 
VIP Call Girls Indore Kirti 💚😋 9256729539 🚀 Indore Escorts
VIP Call Girls Indore Kirti 💚😋  9256729539 🚀 Indore EscortsVIP Call Girls Indore Kirti 💚😋  9256729539 🚀 Indore Escorts
VIP Call Girls Indore Kirti 💚😋 9256729539 🚀 Indore Escortsaditipandeya
 
Night 7k to 12k Navi Mumbai Call Girl Photo 👉 BOOK NOW 9833363713 👈 ♀️ night ...
Night 7k to 12k Navi Mumbai Call Girl Photo 👉 BOOK NOW 9833363713 👈 ♀️ night ...Night 7k to 12k Navi Mumbai Call Girl Photo 👉 BOOK NOW 9833363713 👈 ♀️ night ...
Night 7k to 12k Navi Mumbai Call Girl Photo 👉 BOOK NOW 9833363713 👈 ♀️ night ...aartirawatdelhi
 
All Time Service Available Call Girls Marine Drive 📳 9820252231 For 18+ VIP C...
All Time Service Available Call Girls Marine Drive 📳 9820252231 For 18+ VIP C...All Time Service Available Call Girls Marine Drive 📳 9820252231 For 18+ VIP C...
All Time Service Available Call Girls Marine Drive 📳 9820252231 For 18+ VIP C...Arohi Goyal
 
VIP Call Girls Tirunelveli Aaradhya 8250192130 Independent Escort Service Tir...
VIP Call Girls Tirunelveli Aaradhya 8250192130 Independent Escort Service Tir...VIP Call Girls Tirunelveli Aaradhya 8250192130 Independent Escort Service Tir...
VIP Call Girls Tirunelveli Aaradhya 8250192130 Independent Escort Service Tir...narwatsonia7
 
Call Girls Darjeeling Just Call 9907093804 Top Class Call Girl Service Available
Call Girls Darjeeling Just Call 9907093804 Top Class Call Girl Service AvailableCall Girls Darjeeling Just Call 9907093804 Top Class Call Girl Service Available
Call Girls Darjeeling Just Call 9907093804 Top Class Call Girl Service AvailableDipal Arora
 
Call Girls Nagpur Just Call 9907093804 Top Class Call Girl Service Available
Call Girls Nagpur Just Call 9907093804 Top Class Call Girl Service AvailableCall Girls Nagpur Just Call 9907093804 Top Class Call Girl Service Available
Call Girls Nagpur Just Call 9907093804 Top Class Call Girl Service AvailableDipal Arora
 
Call Girls Siliguri Just Call 9907093804 Top Class Call Girl Service Available
Call Girls Siliguri Just Call 9907093804 Top Class Call Girl Service AvailableCall Girls Siliguri Just Call 9907093804 Top Class Call Girl Service Available
Call Girls Siliguri Just Call 9907093804 Top Class Call Girl Service AvailableDipal Arora
 
Call Girls Coimbatore Just Call 9907093804 Top Class Call Girl Service Available
Call Girls Coimbatore Just Call 9907093804 Top Class Call Girl Service AvailableCall Girls Coimbatore Just Call 9907093804 Top Class Call Girl Service Available
Call Girls Coimbatore Just Call 9907093804 Top Class Call Girl Service AvailableDipal Arora
 
Top Rated Bangalore Call Girls Mg Road ⟟ 8250192130 ⟟ Call Me For Genuine Sex...
Top Rated Bangalore Call Girls Mg Road ⟟ 8250192130 ⟟ Call Me For Genuine Sex...Top Rated Bangalore Call Girls Mg Road ⟟ 8250192130 ⟟ Call Me For Genuine Sex...
Top Rated Bangalore Call Girls Mg Road ⟟ 8250192130 ⟟ Call Me For Genuine Sex...narwatsonia7
 
Best Rate (Hyderabad) Call Girls Jahanuma ⟟ 8250192130 ⟟ High Class Call Girl...
Best Rate (Hyderabad) Call Girls Jahanuma ⟟ 8250192130 ⟟ High Class Call Girl...Best Rate (Hyderabad) Call Girls Jahanuma ⟟ 8250192130 ⟟ High Class Call Girl...
Best Rate (Hyderabad) Call Girls Jahanuma ⟟ 8250192130 ⟟ High Class Call Girl...astropune
 

Recently uploaded (20)

Artifacts in Nuclear Medicine with Identifying and resolving artifacts.
Artifacts in Nuclear Medicine with Identifying and resolving artifacts.Artifacts in Nuclear Medicine with Identifying and resolving artifacts.
Artifacts in Nuclear Medicine with Identifying and resolving artifacts.
 
Call Girls Kochi Just Call 9907093804 Top Class Call Girl Service Available
Call Girls Kochi Just Call 9907093804 Top Class Call Girl Service AvailableCall Girls Kochi Just Call 9907093804 Top Class Call Girl Service Available
Call Girls Kochi Just Call 9907093804 Top Class Call Girl Service Available
 
Vip Call Girls Anna Salai Chennai 👉 8250192130 ❣️💯 Top Class Girls Available
Vip Call Girls Anna Salai Chennai 👉 8250192130 ❣️💯 Top Class Girls AvailableVip Call Girls Anna Salai Chennai 👉 8250192130 ❣️💯 Top Class Girls Available
Vip Call Girls Anna Salai Chennai 👉 8250192130 ❣️💯 Top Class Girls Available
 
(👑VVIP ISHAAN ) Russian Call Girls Service Navi Mumbai🖕9920874524🖕Independent...
(👑VVIP ISHAAN ) Russian Call Girls Service Navi Mumbai🖕9920874524🖕Independent...(👑VVIP ISHAAN ) Russian Call Girls Service Navi Mumbai🖕9920874524🖕Independent...
(👑VVIP ISHAAN ) Russian Call Girls Service Navi Mumbai🖕9920874524🖕Independent...
 
Call Girl Coimbatore Prisha☎️ 8250192130 Independent Escort Service Coimbatore
Call Girl Coimbatore Prisha☎️  8250192130 Independent Escort Service CoimbatoreCall Girl Coimbatore Prisha☎️  8250192130 Independent Escort Service Coimbatore
Call Girl Coimbatore Prisha☎️ 8250192130 Independent Escort Service Coimbatore
 
Russian Call Girls in Delhi Tanvi ➡️ 9711199012 💋📞 Independent Escort Service...
Russian Call Girls in Delhi Tanvi ➡️ 9711199012 💋📞 Independent Escort Service...Russian Call Girls in Delhi Tanvi ➡️ 9711199012 💋📞 Independent Escort Service...
Russian Call Girls in Delhi Tanvi ➡️ 9711199012 💋📞 Independent Escort Service...
 
♛VVIP Hyderabad Call Girls Chintalkunta🖕7001035870🖕Riya Kappor Top Call Girl ...
♛VVIP Hyderabad Call Girls Chintalkunta🖕7001035870🖕Riya Kappor Top Call Girl ...♛VVIP Hyderabad Call Girls Chintalkunta🖕7001035870🖕Riya Kappor Top Call Girl ...
♛VVIP Hyderabad Call Girls Chintalkunta🖕7001035870🖕Riya Kappor Top Call Girl ...
 
Call Girls Horamavu WhatsApp Number 7001035870 Meeting With Bangalore Escorts
Call Girls Horamavu WhatsApp Number 7001035870 Meeting With Bangalore EscortsCall Girls Horamavu WhatsApp Number 7001035870 Meeting With Bangalore Escorts
Call Girls Horamavu WhatsApp Number 7001035870 Meeting With Bangalore Escorts
 
Chandrapur Call girls 8617370543 Provides all area service COD available
Chandrapur Call girls 8617370543 Provides all area service COD availableChandrapur Call girls 8617370543 Provides all area service COD available
Chandrapur Call girls 8617370543 Provides all area service COD available
 
(Rocky) Jaipur Call Girl - 9521753030 Escorts Service 50% Off with Cash ON De...
(Rocky) Jaipur Call Girl - 9521753030 Escorts Service 50% Off with Cash ON De...(Rocky) Jaipur Call Girl - 9521753030 Escorts Service 50% Off with Cash ON De...
(Rocky) Jaipur Call Girl - 9521753030 Escorts Service 50% Off with Cash ON De...
 
VIP Call Girls Indore Kirti 💚😋 9256729539 🚀 Indore Escorts
VIP Call Girls Indore Kirti 💚😋  9256729539 🚀 Indore EscortsVIP Call Girls Indore Kirti 💚😋  9256729539 🚀 Indore Escorts
VIP Call Girls Indore Kirti 💚😋 9256729539 🚀 Indore Escorts
 
Night 7k to 12k Navi Mumbai Call Girl Photo 👉 BOOK NOW 9833363713 👈 ♀️ night ...
Night 7k to 12k Navi Mumbai Call Girl Photo 👉 BOOK NOW 9833363713 👈 ♀️ night ...Night 7k to 12k Navi Mumbai Call Girl Photo 👉 BOOK NOW 9833363713 👈 ♀️ night ...
Night 7k to 12k Navi Mumbai Call Girl Photo 👉 BOOK NOW 9833363713 👈 ♀️ night ...
 
All Time Service Available Call Girls Marine Drive 📳 9820252231 For 18+ VIP C...
All Time Service Available Call Girls Marine Drive 📳 9820252231 For 18+ VIP C...All Time Service Available Call Girls Marine Drive 📳 9820252231 For 18+ VIP C...
All Time Service Available Call Girls Marine Drive 📳 9820252231 For 18+ VIP C...
 
VIP Call Girls Tirunelveli Aaradhya 8250192130 Independent Escort Service Tir...
VIP Call Girls Tirunelveli Aaradhya 8250192130 Independent Escort Service Tir...VIP Call Girls Tirunelveli Aaradhya 8250192130 Independent Escort Service Tir...
VIP Call Girls Tirunelveli Aaradhya 8250192130 Independent Escort Service Tir...
 
Call Girls Darjeeling Just Call 9907093804 Top Class Call Girl Service Available
Call Girls Darjeeling Just Call 9907093804 Top Class Call Girl Service AvailableCall Girls Darjeeling Just Call 9907093804 Top Class Call Girl Service Available
Call Girls Darjeeling Just Call 9907093804 Top Class Call Girl Service Available
 
Call Girls Nagpur Just Call 9907093804 Top Class Call Girl Service Available
Call Girls Nagpur Just Call 9907093804 Top Class Call Girl Service AvailableCall Girls Nagpur Just Call 9907093804 Top Class Call Girl Service Available
Call Girls Nagpur Just Call 9907093804 Top Class Call Girl Service Available
 
Call Girls Siliguri Just Call 9907093804 Top Class Call Girl Service Available
Call Girls Siliguri Just Call 9907093804 Top Class Call Girl Service AvailableCall Girls Siliguri Just Call 9907093804 Top Class Call Girl Service Available
Call Girls Siliguri Just Call 9907093804 Top Class Call Girl Service Available
 
Call Girls Coimbatore Just Call 9907093804 Top Class Call Girl Service Available
Call Girls Coimbatore Just Call 9907093804 Top Class Call Girl Service AvailableCall Girls Coimbatore Just Call 9907093804 Top Class Call Girl Service Available
Call Girls Coimbatore Just Call 9907093804 Top Class Call Girl Service Available
 
Top Rated Bangalore Call Girls Mg Road ⟟ 8250192130 ⟟ Call Me For Genuine Sex...
Top Rated Bangalore Call Girls Mg Road ⟟ 8250192130 ⟟ Call Me For Genuine Sex...Top Rated Bangalore Call Girls Mg Road ⟟ 8250192130 ⟟ Call Me For Genuine Sex...
Top Rated Bangalore Call Girls Mg Road ⟟ 8250192130 ⟟ Call Me For Genuine Sex...
 
Best Rate (Hyderabad) Call Girls Jahanuma ⟟ 8250192130 ⟟ High Class Call Girl...
Best Rate (Hyderabad) Call Girls Jahanuma ⟟ 8250192130 ⟟ High Class Call Girl...Best Rate (Hyderabad) Call Girls Jahanuma ⟟ 8250192130 ⟟ High Class Call Girl...
Best Rate (Hyderabad) Call Girls Jahanuma ⟟ 8250192130 ⟟ High Class Call Girl...
 

ETAS_MCQ_16 dermatological drugs

  • 1. Dermatological drugs 1) Which of the following antiviral agents is NOT phosphorylated by viral thymidine kinase? A. Valacyclovir B. Gancyclovir C. CidofovirCorrect Choice D. Famciclovir E. Acyclovir Cidofovir is a nucleotide analogue antiviral agent. It does not require phosphorylation by virus, but is converted by host cell kinases to a diphosphate 2) Which biologic agent is administered intramuscularly? A. Infliximab B. Efalizumab C. Etanercept D. AlefaceptCorrect Choice E. None of the above Alefacept is given intramuscularly 3) In addition to minocycline, which of the following drugs has been associated with drug-induced lupus erythematosus-like syndrome? A. Rifampin B. Hydralazine Correct Choice C. Itraconazole D. Amiodarone E. Doxycycline Drugs associated with drug-induced SLE include minocycline, hydralazine, procainamide, isonaizid, penicillamine, and anti-convulsants 4) One of the main concerns for prescribing azathioprine to a gout patient with a normal level of thiopurine methyltransferase on chronic allopurinol is: A. Patient may develp photosensitivity B. Azathioprine does not work C. Allopurinol does not work D. Patient may become pancytopenicCorrect Choice E. No concerns Allopurinol inhibits xanthine oxidase, an enzyme that catabolizes azathioprine. In patients concurrently taking bothe medications, the allopurinol shunts more 6-MP from the xanthine oxidase catabolic pathway to the hypoxanthine-guanine phosphoribosyltransferase(HGPRT)anabolic pathway, creating an excess of purine analogs. This in turn may lead to to excessive immunosuppression and risk of pancytopenia 1
  • 2. 5) A patient presents with a likely fixed drug eruption. Her medications include glyburide, lisinopril, hydrochlorthiazide, and aspirin, as well as an over-the-counter laxative. Which is the most likely culprit? A. LaxativeCorrect Choice B. Hydrochlorthiazide C. Lisinopril D. Aspirin E. Glyburide Phenopthalein, found in laxatives, is a known cause of fixed drug eruptions 6) Fomivirsen is a single-stranded antisense oligonucleotide FDA-approved for the treatment of: A. HSV infection B. EBV infection C. HPV infection D. CMV infectionCorrect Choice E. HIV infection Fomiversen (Vitraene) represents a new class of therapeutic agents known as antisense drugs. It is indicated for the treatment of CMV retinitis in patients with AIDS. The most common adverse effects are ocular inflammation and increases in intraocular pressure 7) Which antihistamine has suppressor T-cell inhibitory activity? A. Promethazine B. Fexofenadine C. Doxepin D. Cromolyn sodium E. CimetidineCorrect Choice Cimetidine is an H2 antihistamine that has suppressor T-cell inhibitory activity, by competitively blocking their H2 receptors. Immunomodulatory effects are useful for treating mucocutaneous candidiasis, verruca vulgaris, and condyloma acuminata 8) A 59 year-old woman is diagnosed with acute myelogenous leukemia. Induction chemotherapy is initiated. A few days later the patient develops tender erythematous plaques on her face. What is the most likely culprit drug? A. Methotrexate B. Cyclophosphamide C. CytarabineCorrect Choice D. 5-fluorouracil E. Hydroxyurea 2
  • 3. The most likely diagnosis is neutrophilic eccrine hidradenititis. Cytarabine is the usual offending agent 9) Which drug has mucocutaneous side effects which can include stomatitis, chelitis, lichen planus- like eruptions, and pityriasis rosea-like eruptions? A. GoldCorrect Choice B. Thalidomide C. Colchicine D. Potassium iodide E. None of the above Mucocutaneous side effects, which are more common with injectable gold, include stomatitis, cheilitis, lichen planus-like eruptions, and pityriasis rosea-like eruptions 10) Which of the following is not a side effect of bleomycin? A. Flagellate hyperpigmentation B. Serpentine supravenous hyperpigmentationCorrect Choice C. Pulmonary fibrosis D. Acrosclerosis E. Erythromelalgia All of the above are side effects of bleomycin except serpentine supravenous hyperpigmentation. This is a side-effect of 5-fluorouracil 11) Side effects from this antihistamine include gynecomastia, impotence, and loss of libido: A. Fexofenadine B. Promethazine C. Cyproheptadine D. CimetidineCorrect Choice E. Doxepin Cimetidine, an H2 antihistamine, also competitively inhibits dihydrotestosterone at the androgen receptor site, with resultant antiandrogen side effects including gynecomastia, impotence, and loss of libido 12) Which drug is known to cause an SCLE-like eruption? A. Barbituates B. Captopril C. Minocycline D. Terbinafine Correct Choice E. Furosemide SCLE-like reaction are known to occur most likely from glyburide, griseofulvin, hydrochlorothiazide, penicillamine, piroxicam, and terbinafine 3
  • 4. 13) Which of the following drugs may cause acute generalized exanthematous pustulosis? A. Mercury B. All of these answers are correctCorrect Choice C. Cefazolin D. Ampicillin E. Azithromycin Acute generalized exanthematous pustulosis (AGEP) has been associated with beta-lactam antibiotics, macrolide antibiotics, cephalosporins, and mercury. 14) What is the most likely congenital defect associated with isotretinoin therapy? A. Ventral septal defect B. Cranial synostosisCorrect Choice C. Spina bifida D. Phocomelia E. Atrial septal defect Isotretinoin is thought to cause congenital defects by interfering with neural crest development. The most likely congenital defect is cranial synostosis 15) What drug is known to cause scotomas? A. Gold B. Dapsone C. Quinacrine D. Isotretinoin E. Chloroquine Correct Choice True retinopathy is associated with “bull’s eye” pigment deposition, central scotoma, and diminished visual acuity. Chloroquine is at greatest risk of causing retinopathy. Quinacrine is not associated with the risk of retinopathy 16) Finasteride is a specific inhibitor of: A. Aromatase B. Type II 5 alpha reductaseCorrect Choice C. Dihydrotesterone reductase D. Testosterone synthetase E. Type I 5 alpha reductase Finasteride, a type II 5 alpha reductase inhibitor, given as a 1mg tablet daily, is effective in preventing further hair loss and in increasing the hair counts to the point of cosmetically appreciatable results in men ages 18 to 41 with mild to moderate hair loss at the vertex, in the anterior midscalp, and the frontal region 4
  • 5. 17) Which of the following pairings of antifungal agents and their mechanisms of action is NOT correct? A. Fluconazole: Inhibits squalene epoxidaseCorrect Choice B. Ketoconazole: Blocks conversion of lanosterol to ergosterol C. Itraconazole: Inhibits 14-alpha-demethylase D. Griseofulvin: Disrupts microtubule mitotic spindle formation E. Terbinafine: Inhibits squalene epoxidase Terbinafine inhibits squalene epoxidase and blocks the biosynthesis of ergosterol, a sterol essential to the integrity of fungal cell membranes. Itraconazole inhibits 14-alpha-demethylase, blocking lanosterol conversion to ergosterol. Griseofulvin disrupts microtubule mitotic spindle formation causing metaphase arrest. Ketoconazole has a mechanism of action similar to itraconazole. Fluconazole also inhibits 14-alpha-demethylase, not squalene epoxidase 18) Vascular leak syndrome has been associated with which chemotherapeutic agent? A. Bleomycin B. Cytarabine C. Denileukin diftitoxCorrect Choice D. Methotrexate E. Interferon Denileukin difitox (brand name Ontak) is approved for the treatment of cutaneous T-cell lymphoma. Denileukin difitox is a fusion protein composed from a portion of IL-2 with diphtheria toxin. The chemotherapy is taken up by cells expressing high-affinity IL-2 receptors. Capillary leak syndrome presenting with hypotension, edema, pleural effusions, and weight gain caused by fluid retention has been reported in patients receiving denileukin difitox 19) Methotrexate toxicity with myelosuppression is treated with what agent? A. Folic acid B. Vitamin E C. Folinic acid Correct Choice D. Cimetidine E. Oral methylene blue Leukovorin, or folinic acid, is able to bypass dihydrofolate reductase in the cell division pathway. It is administered under conditions of methotrexate-induced myelosuppression. Leukovorin is also available as an oral form, and may be administered as a continual part of methotrexate therapy, instead of folic acid. It is now believed that neither folic acid nor folinic acid inhibit the efficacy of methotrexate’s antipsoriatic actions 20) Which one of the following antifungals requires an acidic environment for its absorption? A. terbinafine B. fluconazole C. itraconazoleCorrect Choice 5
  • 6. D. griseofulvin E. amphotericin B Itraconazole is a triazole whose mechanism, like the other azoles, is inhibition of cytochrome P-450 enzyme lanosterol 14-alpha demethylase, with resultant inhibition of lanosterol conversion to ergosterol. Ergosterol is an essential component of fungal cell membranes. Itraconazole is a weak base, which is highly lipophilic and virtually insoluble in water. It is ionized only at a low pH. Griseofulvin is administered in divided doses for the treatment of tinea capitus. Its absorption is optimized when given with a fatty food. Oral fluconazole is very well absorbed when given orally with over 90% bioavailability. Terbinafine, though highly lipophilic, has 70 to 80% absorption, when administered orally, with a bioavailability not significantly impacted by food intake 21) Bone marrow suppression can occur more often in individuals taking azathioprine with genetically low allele activity of what enzyme? A. Dihydrofolate reductase B. Inosine monophosphate dehydrogenase C. Glucose-6-phosphatase D. Epoxide hydroxylase E. Thiopurine methyltransferase Correct Choice NEEDS EXPLANATIONS 22) What antifungal is the best choice for a patient with mucocutaneous candidiasis who is currently taking antacids? A. Griseofulvin B. Fluconazole Correct Choice C. Terbinafine D. Itraconazole E. Ketoconazole Ketoconazole and itraconazole require an acidic environment. Fluconazole, however, does not require an acidic environment and can work safely and effectively in patients taking antacids, which can raise gastric pH levels 23) The treatment of choice for scabies in pregnant women is: A. Malathion B. Thiabendazole C. 1% permethrin D. Precipitated sulfur 6% Correct Choice E. Ivermectin NEEDS EXPLANATIONS 24) Approximately what percentage of patients with drug hypersensitivity syndrome will have liver function test abnormalities? A. Close to 100% 6
  • 7. B. 25% C. Less than 10% D. 75% E. 50% Correct Choice Drug hypersensitivity syndrome is characterized by fever, skin eruption and internal organ involvement. Drugs associated with drug hypersensitivity syndrome include sulfonamindes, dapsone, anticonvulsants (carbamezapine, phenobarbitol, lamotrigine), anti-retrovirals (ritonovir, nevirapine) and minocycline. Approximately 50% of patients will have abnormal liver enzymes 25) Weekly CD4 T-cell counts are recommended for psoriasis patients treated with which biologic agent? A. Etanercept B. None of the above C. Efalizumab D. AlefaceptCorrect Choice E. Infliximab Alefacept eliminates activated memory T-cells, so weekly CD4 T-cell counts are recommended 26) The antihistamine with strong H1 and H2 blockade is: A. Cyproheptadine B. Cimetidine C. Cetirizine D. Doxepin Correct Choice E. Chlorpheniramine Doxepin, a tricyclic antidepressant, has H1 and H2 antihistamine activity 27) Which of the following statements regarding retinoids is true? A. Tretinoin and isotretinoin are second-generation synthetic retinoids B. Second-generation retinoids are polyaromatic retinoids C. Bexarotene is a third-generation retinoidCorrect Choice D. Etretinate is a first-generation retinoid E. Tazarotene is a fourth-generation selective retinoid First-generation retinoids are tretinoin (all-trans RA) and isotretinoin (13-cis RA). Second- generation synthetic retinoids are etretinate, which was replaced by its metabolite acitretin. Third- generation (polyaromatic retinoids) include the arotinoids, tazarotene, adapalene, and bexarotene 28) Which of the following antifungal agent works by way of inhibiting ergosterol synthesis by blocking squalene epoxidation: A. Terbinifine NaftifineCorrect Choice B. Itraconazole 7
  • 8. C. Naftifine D. Itraconazole and Terbinifine E. Terbinifine Terbinafine and Naftifine work by way of inhibiting ergosterol synthesis by blocking squalene epoxidation (B&C). Itraconazole works by inhibiting ergosterol synthesis by blocking Lanosterol 14- alpha demthylase 29) At standard dosages, which of the following is fungicidal? A. TerbinafineCorrect Choice B. Itraconazole C. Ketoconazole D. Griseofulvin E. Fluconazole Terbinafine block ergosterol synthesis early in the synthetic pathway by inhibitins squalene epoxidase. Squalene then accumulates within fungal cells and discupts cell membranes. At standard dosaging, it is believed to be fungicidal. The other choices are fungistatic 30) Which drug has been associated with cholestatic hepatitis? A. Clindamycin B. ErythromycinCorrect Choice C. Doxycycline D. Rifampin E. Ampicillin The estolate form of erythromycin has been associated with cholestatic hepatitis 31) Clofazamine hyperpigmentation has been described as: A. Red-brown hyperpigmentation within skin lesions Correct Choice B. Slate gray-purple hyperpigmentation in a photo-exposed distribution C. Yellow discoloration of the skin, sclera D. Dark hyperpigmented streaks in the nails E. Blue-gray hyperpigmentation over the anterior shins, palate, ears Clofazamine can induce red-brown hyperpigmentation within skin lesions of patients with Hansen’s disease 32) Of the new biologic therapies for psoriasis, which agent requires weekly CD4 T-cell count monitoring? A. Adalimumab B. Efalizumab C. Alefacept Correct Choice 8
  • 9. D. Infliximab E. Etanercept Alefacept is a human LFA-3/IgG fusion protein, which blocks LFA-3 on antigen presenting cells from interacting with CD-2 on T-cells, preventing T-cell stimulation. Alefacept also eliminates activated memory-effector T-cells, so weekly CD4 T-cell counts are required 33) Which drug has recently been shown to be of use in chemoprevention of squamous cell carcinoma in recessive dystrophic epidermolysis bullosa: A. Methotrexate B. Mycophenolate mofetil C. Cyclosporine D. IsotretinoinCorrect Choice E. Topical tar Despite low therapeutic responses of advanced cancers to retinoids, these drugs appear to have a promising role in chemoprevention. Patients with oral leukoplakia, actinic keratoses, arsenic keratoses, and Bowen's disease can benefit from retinoid therapy 34) The mechanism of action of podophyllin most closely resembles that of what other drug listed below? A. Dactinomycin B. Permethrin C. 5-fluorouracil D. Chlorambucil E. Colchicine Correct Choice Both podophyllin and colchicine have antimitotic activity. They bind to tubulin dimers, interfering with mitotic spindle and microtubule assembly 35) Raynaud’s phenomenon is a potential adverse effect of what chemotherapeutic agent? A. Doxorubicin B. Hydroxyurea C. Vinca alkaloids D. 5-fluorouracil E. Bleomycin Correct Choice Bleomycin use has been associated with Raynaud’s phenomenon occurring in digits treated with intralesional therapy for periungual and plantar warts 36) What antihistamine can cause gynecomastia, impotence, and loss of libido? A. Doxepin B. Cyproheptadine C. Cimetidine Correct Choice 9
  • 10. D. Fexofendine E. Chlorpheniramine Cimetidine competitively inhibits dihydrotestosterone at the androgen receptor site and can exhibit anti-androgen effects. 37) Methemoglobinemia is an adverse reaction to treatment with which agent? A. Plaquenil B. DapsoneCorrect Choice C. Methotrexate D. Cyclophosphamide E. Azathioprine Dapsone is a lipid-soluble sulfone drug that is used widely in dermatology in a variety of conditions including dermatitis herpetiformis, leprosy, and neutrophilic dermatoses. Adverse effects from dapsone are both pharmacologic and idiosyncratic and include hemolytic anemia, methemoglobinemia, agranulocytosis, hypersensitivity syndrome and neuropathy. Of these, the first two are pharmacologic and anticipated, to some degree, in most patients treated with dapsone. However, the magnitude of toxicity varies greatly among individuals on the drug. Methemoglobinemia is the formation of methemoglobin in the blood, which has a decreased oxygen- carrying capacity compared with hemoglobin and can result in cyanosis. The reaction is related to the N-hydroxy metabolites of dapsone, which are potent oxidants. G6PD-deficient individuals are more susceptible to oxidative stresses, including those from dapsone metabolites, and a baseline G6PD level is recommended prior to initiation of dapsone therapy. In the event of emergent methemoglobinemia, oral methylene blue (100-300 mg/day) can be used to decrease methemoglobin levels. However, if the patient is G6PD deficient, this strategy is ineffective 38) Which of the following statements is NOT true regarding the categories for safety of drug use in pregnancy? A. Drugs are category D if controlled studies show risk to human fetus, but in some instances benefits may outweigh risks B. Drugs are category A if controlled studies in humans show no risk to the fetus C. Drugs are category B if controlled human studies show no risk to the fetus but may show risk to animals, or if no risk has been shown in animal studies but no human studies have been conducted D. Category X drugs are contraindicated in pregnancy E. Drugs are category C if risk to the human fetus has been demonstrated, but animal studies are equivocalCorrect Choice Drugs are category C if risk to the human fetus cannot be ruled out, studies are lacking, or animal studies are equivocal. Drugs for which risk to the human fetus has been demonstrated are pregnancy category D 39) The risk of pseudotumor cerebri in patients taking isotretinoin is increased by: A. Dehydration B. Comorbid affective disorder C. Doses higher than 1.0 mg/kg/day D. Concomitant use of tetracycline Correct Choice 10
  • 11. E. Concomitant use of TMP-SMX The risk of pseudotumor cerebri is increased in patients on isotretinoin and a tetracycline 40) Neutrophilic eccrine hidradenitis is associated most often with which chemotherapeutic agent? A. Vinca alkaloids B. 5-fluorouracil C. Cytarabine Correct Choice D. Dactinomycin E. Doxorubicin Neutrophilic eccrine hidradenitis occurs in neutropenic patients with malignancies, usually acute myelogenous leukemia. Cytarabine is the most commonly associated medication 41) Which of the following statements regarding antifungal medications is TRUE? A. Terbinafine is fungistatic B. Griseofulvin is safe for patients with variegate porphyria C. Griseofulvin is a cytochrome P450 3A4 inhibitor D. Ketoconazole is fungicidal E. Ketoconazole has been associated with gynecomastiaCorrect Choice Medications that can precipitate acute attacks in variegate porphyria include barbiturates, estrogen, griseofulvin, sulfonamides and ethanol. Ketoconazole can produce impotence and gynecomastia by interfering with androgen synthesis. Ketoconazole inhibits cytochrome (CYP) P450 and most concerning, can rarely cause fulminant hepatitis. Griseofulvin induces CYP P450, not inhibits it. Terbinafine is fungicidal along with amphotericin B. Ketoconazole is fungistatic 42) For which of the following medications is abdominal cramping and watery diarrhea a very common side effect that may limit treatment? A. ColchicineCorrect Choice B. Chlorambucil C. Potassium iodide D. Gold E. Thalidomide Colchicine is an alkaloid with antimitotic activity that is used in dermatology for its effects on neutrophils. The most common side effect from colchicine use is gastrointestinal distress with abdominal cramping and watery diarrhea 43) The combination of ethanol and acitretin is potentially problematic because: A. Ethanol inhibits the cytochrome p450 system B. Ethanol exacerbates the cheilitis caused by acitretin C. Ethanol promotes the conversion of acitretin to etretinateCorrect Choice D. Ethanol promotes the metabolism of acitretin 11
  • 12. E. Acitretin increases the toxicity of ethanol Acitretin is an synthetic retinoid with affinity to the retinoic acid receptor (RAR). It can modulate the proliferation and differentiation of epidermal keratinocytes. The concurrent injestion of ethanol and alcohol increases the formation of etretinate. Unlike acitretin, etretinate is more lipophilic and accumulates in the fat. It therefore has a much longer elimination half-life, estiimated at 120 days 44) The combination of ethanol and acitretin is potentially problematic because: A. Ethanol inhibits the cytochrome p450 system B. Ethanol exacerbates the cheilitis caused by acitretin C. Ethanol promotes the conversion of acitretin to etretinateCorrect Choice D. Ethanol promotes the metabolism of acitretin E. Acitretin increases the toxicity of ethanol Acitretin is an synthetic retinoid with affinity to the retinoic acid receptor (RAR). It can modulate the proliferation and differentiation of epidermal keratinocytes. The concurrent injestion of ethanol and alcohol increases the formation of etretinate. Unlike acitretin, etretinate is more lipophilic and accumulates in the fat. It therefore has a much longer elimination half-life, estiimated at 120 days 45) Which of the following supplements is most likely to decrease hemolysis associated in patients taking dapsone? A. Vitamin A B. Folic acid C. Vitamin ECorrect Choice D. Vitamin B6 E. Vitamin D Adverse effects from dapsone are both pharmacologic and idiosyncratic and include hemolytic anemia, methemoglobinemia, agranulocytosis, hypersensitivity syndrome and neuropathy. Of these, the first two are pharmacologic and anticipated, to some degree, in most patients treated with dapsone. However, the magnitude of toxicity varies greatly among individuals on the drug. Methemoglobinemia is the formation of methemoglobin in the blood, which has a decreased oxygen- carrying capacity compared with hemoglobin and can result in cyanosis. The reaction is related to the N-hydroxy metabolites of dapsone, which are potent oxidants. G6PD-deficient individuals are more susceptible to oxidative stresses, including those from dapsone metabolites, and a baseline G6PD level is recommended prior to initiation of dapsone therapy. Vitamin E (800 IU/day) has been shown to provide a small amount of protection against methemoglobinemia and hemolysis, however, the clinical benefit of this strategy is unclear 46) Which of the following has been associated with a lichenoid drug eruption? A. Acetaminophen B. Sulfasalazine C. Hydrochlorothiazide Correct Choice D. Erythromycin E. Nicotinamide Lichen-planus-like (lichenoid) drug eruptions have been reported with: antimalarials, ?-blockers, captopril, gold, penicillamine, HCTZ, NSAIDs. Lichenoid drug reactions are often photodistributed 12
  • 13. 47) Which of the following is a side effect of hydroquinone cream? A. Telangiectasia B. Photosensitivity C. Atrophy D. OchronosisCorrect Choice E. Tachyphylaxis Exogenous ochronosis is an uncommon complication of irreversible pigmentation due to overuse of topical hydroquinone (1,4 dihydroxybenzene). Hydroquinone acts to by melanocyte pigment production by auto-oxidation of melanin, tyrosinase and phenol oxidases. 48) Which antiparasitic agent is highly flammable? A. MalathionCorrect Choice B. Lindane C. Precipitated sulfur D. Thiabendazole E. Permethrin Malathion, an organophosphate cholinesterase inhibitor used to treat scabies and head lice, is flammable 49) Cyclosporine forms a complex to directly interfere with activation of what calcium-dependent protein? A. Calcineurin Correct Choice B. Cyclophilin C. Cytochrome P-450 D. Calmodulin E. NFAT-1 Cyclosporine inhibits calcineurin, a phosphatase activated in the presence of calmodulin and calcium, by cyclophilin. Cyclosporine forms a complex with cyclophilin, blocking its ability to activate calcineurin, and thus preventing calcineurin from phosphorylating NFAT-1, a transcription factor. NFAT-1, when phosphorylated can travel to the nucleus of cells, initiate IL-2 production, and stimulate T-cell proliferation 50) What is the target antigen for rituximab? A. CD4 B. CD22 C. CD7 D. CD20Correct Choice E. CD8 Rituximab is an anti-CD20 monoclonal antibody. CD20 is a B-cell marker and is used to treat Non- Hodgkins B-cell lymphoma. It is also approved for the treatment of rheumatoid arthritis 13
  • 14. 51) What medication's mechanism of action is via suppression of the halide-myeloperoxidase system? A. Hydroxyurea B. Melphalan C. Quinicrine D. DapsoneCorrect Choice E. Cyclophosphamide Dapsone inhibits the neutrophil halide-myeloperoxidase system which results in an impaired respiratory burst and subsequent tissue damage 52) A 56 year-old man presents with blue-gray discoloration on his face, ears, and dorsal hands. What is the most likely offending agent? A. Clofazimine B. Quinacrine C. Minocycline D. AmiodaroneCorrect Choice E. Chloroquine The patient presents with blue-gray discoloration in sun-exposed areas. The most likely offending agent is amiodarone. Blue-gray discoloration from minocycline usually occurs on legs. Chloroquine usually causes blue-gray discoloration in the sclerae, teeth, buccal mucosa, nail beds, and pretibial areas. Quinacrine causes yellow discoloration of skin and conjunctiva. CLofazimine usualy causes a red-brown discoloration 53) Painful periungual pyogenic granulomas have been associated with what medication? A. Indinavir Correct Choice B. Ketoconazole C. Doxycycline D. Valacyclovir E. Tazarotene Periungual pyogenic granulomas and painful paronychial eruptions have been reported in association with various anti-HIV medications including, indinavir, zidovudine, and lamivudine 54) Which drug may increase levels of digoxin? A. Amoxicillin B. Minocycline C. Cephalexin D. Ciprofloxacin E. ErythromycinCorrect Choice 14
  • 15. Erythromycin inhibits the cytochrome P-450 system, which may result in increased levels of digoxin, among many other drugs 55) A 10 year old child with a seizure disorder develops a morbiliform eruption and elevated LFT’s two weeks after starting Dilantin therapy. As his physician you: A. Discontinue Dilantin and begin Phenobarbital B. Continue Dilantin and treat rash with topical corticosteroids C. Discontinue Dilantin and begin carbamazepine D. Restart Dilantin once the rash resolves E. Discontinue Dilantin and begin valproic acidCorrect Choice Anticonvulsant hypersensitivity syndrome (also drug rash with eosinophilia and systemic symptoms (DRESS0 and dilantin hypersensitivity syndrome) presents with cutaneous eruption accompanied by fever, facial edema, lymphadenopathy, leukocytosis and hepatitis. Cross reactivity is present in all aromatic anticonvulsants including phenytoin, carbamazepine and phenobarbitol. There is no cross reaction with valprioc acid 56) Potentially fatal ventricular arrhythmias can occur with concomitant use of cisapride and: A. Terbinafine B. Atorvostatin C. Astemizole D. Erythromycin Correct Choice E. Digoxin Co-administration of erythromycin with the antihistamines terfenedine and astemizole or the gastrointestinal promobility agent cisapride increases the risk of torsade de pointes and is contraindicated. These drugs are no longer available in the US 57) The agent of choice used to acutely lower methemoglobin levels in patients taking dapsone is: A. Cimetidine B. Vitamin E C. Oral methylene blue Correct Choice D. Homocysteine E. Glucose-6-phosphatase Cimetidine and vitamin E have both been known to provide prophylaxis against methemoglobin formation. G6PD-deficient individuals are at greater risk of hematologic toxicity from dapsone 58) The most specific marker of drug-induced lupus is: A. Anti-La Ab B. ANA C. Anti-ds DNA Ab D. Anti-histone Ab Correct Choice 15
  • 16. E. Anti-Ro Ab Anti-histone Ab is most specific for drug-induced lupus. In addition to minocycline, hydralazine, procainamide, isonaizid (INH), penicillamine and anti-convulsants have been associated with drug- induced lupus-like syndrome 59) Which of the following is the most common adverse effect of Thalidomide therapy? A. Diarrhea B. Hypertension C. Photosensitivity D. Skin discoloration E. SedationCorrect Choice Thalidomide was introduced in the late 1950's as a "safe" sleeping aide. It readily penetrates the CNS, where it exerts a hyposedative effect comparable with barbiturates. By far, the most common adverse effect from thalidomide is sedation, which in many patients may require that primarily night-time doses be utilized 60) Which of the following antiparasitic agents is an organophosphate cholinesterase inhibitor? A. Ivermectin B. Precipitated sulfur C. Thiabendazole D. Lindane E. MalathionCorrect Choice Malathion is an organophosphate cholinesterase inhibitor 61) The treatment of choice for Wegner’s granulomatosis is: A. Methotrexate B. Chlorambucil C. Cyclophosphamide Correct Choice D. Azaithioprine E. Systemic glucocorticosteroids NEEDS EXPLANATIONS 62) Thryoid function tests should be checked before and during therapy with which of the following medications? A. Azathioprine B. Potassium iodideCorrect Choice C. Thalidomide D. Colchicine E. Gold 16
  • 17. The Wolff-Chaifkoff effect, which is the inhibition of thyroid hormone synthesis from excess iodides which block organic iodides from binding in the thyroid, can be observed in patients on potassium iodide therapy. In patients with normal thyroid function, autoregulatory mechanisms allow for appropriate escape from this effect. In patients with impaired autoregulatory mechanisms, the Wolff-Chaikoff effect can lead to hypothyroidism 63) Which of the following is a low sedation metabolite of hydroxyzine? A. Fexofenadine B. Cyproheptadine C. Loratadine D. CetirizineCorrect Choice E. Ranitidine Cetirizine is a second-generation H1 antihistamine that is a low sedation metabolite of hydroxyzine 64) Which of the following statements is not true regarding the tetracycline antiobiotics? A. These antibiotics are effective against Mycoplasma infections B. Tetracycline is more phototoxic than demeclocyclineCorrect Choice C. Tetracyclines are contraindicated in children less than 9 years of age D. Ingestion of zinc salts may impair absorption of tetracycline E. Tetracycline is the most common cause of fixed drug eruption Demeclocycline and doxycycline are the most phototoxic of all the tetracyclines 65) Which of the following is not an ingredient of Castellani's paint? A. Boric Acid B. Resorcinol C. Phenol D. Ethyl acetateCorrect Choice E. Industrial methylated spirit Castellani's paint was named after Sir Aldo Castellani and contains resorcinol, acetone, magenta, phenol, boric acid, industrial methylated spirit, and water. It is fungicidal and bactericidal with local anesthetic effects. It has been used to treat inflammatory tinea cruris, leg ulcers, and acute paronychia 66) What is the difference between podophyllin and podophyllotoxin? A. Essentially the same; they are interchangable in terms of treatment and side effects B. Podophyllin contains kaempherol which is a potent mutagensCorrect Choice C. Podophyllotoxin is a phosphodiesterase inhibitor D. Podophyllotoxin contains quercetin which is a potent mutagens E. Podophyllin reversibly binds tubulin inhibiting cells in metaphase 17
  • 18. Podophyllotoxin, also known as podofilox or Condolox, is a anti-mitotic agent that reversibly binds tubulin, arresting cells in metaphase. It is used topically to treat genital warts. Podophyllin, which has the same mechanism of action, contains kaempero and quercetin which are potent mutagens. Both are derived from the May Apple plant. Cantharin, an antiviral agent derived from the Blister beetle, is a phosphodiesterase inhibitior 67) Which of the following agents is NOT a UVA blocker? A. Dioxybenzone B. Avobenzone C. Red veterinary petrolatum D. Amyl p-dimethylaminobenzoateCorrect Choice E. Dibenzoylmethane Dioxybenzone is one of the benzophenones (as in oxybenzone and sulisobenzone), which are UVA blockers. Amyl p-dimethylaminobenzoate is a UVB blocker. Dibenzoylmethane (avobenzone; Parsol 1789) is a UVA/UVB blocker. Red veterinary petrolatum is a UVA blocker 68) Which of the following class of medications has been associated with acquired brachial dyschromatosis? A. Anti-virals B. HMG-CoA Reductase inhibitors C. Non-Steroidal Anti-inflammatory medications D. ACE-inhibitorsCorrect Choice E. Protease inhibitors Acquired brachial dyschromatosis is a condition described as asymptomatic, gray-brown patches with geographic borders, occasionally interspersed with hypopigmented macules, on the dorsum of the forearms, mostly bilaterally and seen in middle aged women. Epidermal atrophy, basal layer hyperpigmentation, elastosis and angiectases were histopathologic features. An association with Civatte's poikiloderma as well as hypertension and/or antihypertensive drugs, especially ACE- inhibitors, is suggested 69) What is used to reduce bladder toxicity from cyclophosphamide? A. Leukovorin B. Cimetidine C. Folic acid D. Mesna Correct Choice E. Vitamin E Mesna or sodium 2-mercptoethanesulfonate, has been used to reduce bladder toxicity from cyclophophamide 70) The medication most acceptable for usage in patients with renal failure is: A. Oxytetracycline B. Tetracycline 18
  • 19. C. Minocycline D. Demeclocycline E. Doxycycline Correct Choice Renal failure may prolong the half-life of most tetracyclines except doxycycline. Doxycycline is excreted via the GI tract, unlike the other tetracyclines 71) Which of the following statements regarding sunscreens is true? A. Photoallergy has not been reported to benzophenones B. Methyl anthranilate is a UVB absorber C. PABA and its derivates do not cross react with sulfonamides D. Padimate O is a UVB absorberCorrect Choice E. Physical blockers absorb ultraviolet light and convert it to lower energy wavelengths Physical blockers reflect and scatter UV rays, whereas chemical sunscreens absorb UV light and convert the absorbed energy into longer lower energy wavelengths. Methyl anthranilate is a UVA blocker. Padimate O, a PABA derivative, is a UVB blocker. Photoallergy has been reported with increasing frequency to benzophenones. Allergic contact allergy can occur with PABA and its derivatives, which can cross react with azodyes, aniline, procaine, benzocaine, paraphenylenediamine, and sulfonamides 72) A patient with acute diarrhea is prescribed antibiotic treatment for his symptoms. He subsequently suffers from nausea and vomiting after ingesting alcohol. What is the most likely medication he is taking? A. Azithromycin B. Clindamycin C. MetronidazoleCorrect Choice D. Ciprofloxacin E. Penicillin This patient is most likely taking metronidazole for acute diarrhea secondary to giardella. Metronidazole causes antabuse-like reactions with ingestion of alcohol 73) What is the half-life of isotretinoin? A. 50 hours B. 20 hoursCorrect Choice C. 30 days D. 7 hours E. 120 days The half-life of isotretinoin is 20 hours. The half lives of bexarotene, acitretin, etretinate are 7 hours, 50 hours, 120 days respectively 74) Which of the following systemic agents has been shown to be the most effective in the treatment of toenail onychomycosis? 19
  • 20. A. Itraconazole B. Griseofulvin C. Ketoconazole D. TerbinafineCorrect Choice E. Fluconazole Craford et al. reviewed the available literature examining the efficacy of systemic anti-fungals and performed a meta-analysis. Pooled analysis of cure rates at 11 and 12 months suggested that terbinafine was more effective than itraconazole 75) What is the recommended period for contraception after cessation of acitretin therapy in the United States? A. 2 years B. 3 months C. 3 years Correct Choice D. 1 month E. 1 year Etretinate has a prolonged half-life of 80-160 days. The levels may persist up to 3 years in the body. Acitretin can be converted to etretinate in the presence of ethanol 76) The SPF of a sunscreen is based on applying the sunscreen at what concentration? A. 5 mg/cm2 B. 1 mg/cm2 C. 2 mg/cm2Correct Choice D. 4 mg/cm2 E. 3 mg/cm2 A sunscreen SPF is based on using it at a concentration of 2 mg/cm2 which is about 1 ounce or 30 grams for the entire average sized body. It also is about 3-5 grams for the head and neck 77) How long after isotretinoin therapy can one safely begin trying to conceive? A. Two weeks B. One year C. Three years D. Immediately E. One monthCorrect Choice A woman should wait one month before trying to conceive after taking isotretinoin to prevent birth defects. After taking acitretin a woman should wait three years before trying to conceive 78) The following drugs have been implicated in drug-induced subacute cutaneous lupus erythematosus: A. Pravastatin 20
  • 21. B. Terbinafine C. None of the above are correct D. Verapamil E. All the above are correctCorrect Choice All of the above choices have been implicated in drug-induced subacute cutaneous lupus erythematosus 79) Which one of the following sunscreens has an absorption spectrum primarily in the UVA range? A. Cinnamates B. PABA (para-aminobenzoic acid) C. Padimate O D. Octyl Salicylate E. Parsol 1789 (butyl dibenzoylmethane)Correct Choice Avobenzone (Parsol 1789) is primarily a UVA blocker. Photostability of avobenzone may be a problem if it is combined with octyl methoxycinnamate. Salicylates, PABA, Padimate O, and cinnamates are primarily UVB blockers 80) What is the half-life of isotretinoin? A. 50 hours B. 100 days C. 10 hours D. 100 hours E. 20 hoursCorrect Choice The half-life of isotretinoin is 20 hours 81) Tazarotene is what category for safety in pregnancy? A. Category X Correct Choice B. Category A C. Category C D. Category B E. Category D Category X drugs include: acitretin, etretinate, estrogens, finasteride, 5-fluorouracil, flutamide, isotretinoin, methotrexate, stanozolol, thalidomide, and tazarotene 82) Which antiviral agent has been associated with fatal thrombotic thrombocytopenic purpura in AIDS and transplant patients taking high doses? A. Cidofovir B. Penciclovir C. ValacyclovirCorrect Choice 21
  • 22. D. Acyclovir E. Famciclovir Valacyclovir has been associated with severe and even fatal cases of thrombotic thrombocytopenic purpura / HUS syndrome in AIDS and transplant patients taking high doses 83) All of the following statements are true regarding cyclosporin A EXCEPT: A. Adverse effects include hypertrichosis and gingival hyperplasia B. NSAIDs can potentiate renal toxicity when combined with cyclosporine C. The most common eletrolyte abnormalities are hypokalemia and hypermagnesemiaCorrect Choice D. Forms a complex with cyclophilin, blocking its ability to activate calcineurin, thus preventing calcineurin from phosphorylating NFAT-1 E. Metabolized by the hepatic cytochrome P-450 3A4 enzyme system The most common electrolyte abnormalities associated with cyclosporin A are hyperkalemia, hyperuricemia, and hypomagnesemia 84) Which of the following is a true statement about infliximab? A. FDA-approved for the treatment of psoriasis B. Not effective in psoriatic arthritis C. Increases C-reactive protein D. Chimeric monoclonal antibody against TNF-alphaCorrect Choice E. Indicated for mild to moderate psoriasis Infliximab (brand name Remicade) is a chimeric recombinant fusion protein composed of a human TNF-alpha receptor with IgG1-Fc. The immunoglobulin portion is derived from a human constant region and a murine variable region. Infliximab neutralizes both soluble and transmembrane TNF- alpha. This medication has been FDA approved for the treatment of Crohn’s disease, rheumatoid arthritis and ankylosing spondylitis 85) Which of the following drugs is correctly matched to its target enzymes? A. Terbinafine-Cytochrome p450 B. Acyclovir-DNA polymeraseCorrect Choice C. Mycophenolate Mofetil-Phospholipase A2 D. Tacrolimus-Thymidine kinase E. Methotrexate-Inosine monophosphate dehydrogenase Drug/Specific enzyme inhibited Tacrolimus - Calcineurin Methotrexate - Dihydrofolate reductase Mycophenolate mofetil - Inosine monophosphate dyhydogenase Acyclovir - DNA Polymerase Terbinafine - Squalene epoxidase 86) The mechanism of action of mycophenolate mofetil most closely resembles that of what other drug? 22
  • 23. A. Hydroxyurea B. Chlorambucil C. Doxorubicin D. Azathioprine Correct Choice E. Cyclophosphamide Both azaithioprine and mycophenolate mofetil directly interfere with purine synthesis 87) Which biologic agent blocks T-cells from egressing the vasculature and entering the skin? A. EfalizumabCorrect Choice B. Alefacept C. Infliximab D. Etanercept E. None of the above Efalizumab blocks LFA-1 on T-cells from interacting with ICAM-1 on antigen presenting cells, endothelial cells, and cells in the dermis and epidermis. It thus blocks T-cells from egressing the vasculature and entering the skin 88) Which of the following chemotherapeutic agents causes increased growth of eyelashes? A. Mitomycin B. Methotrexate C. Cytarabine D. Interleukin 2 E. Interferon alphaCorrect Choice Interferons can cause increased growth of eyelashes. Trichomegaly has been reported after treatment with interferon-alpha in patients with chronic hepatitis, B-cell lymphoma, chronic granulocytic leukemia, and cutaneous melanoma. Trichomegaly has also been reported in associatoin with latanoprost, minoxidil, cyclosporine, phenytoin, psoralen, and penicillamine 89) For which of the following medications is sedation a very common side effect that may limit treatment? A. Gold B. Colchicine C. Chlorambucil D. ThalidomideCorrect Choice E. Potassium iodide Sedation is a very common side effect of treatment with thalidomide. It is additive with other sedatives, such as alcohol and barbiturates 90) non-pigmenting fixed drug eruption is known to be caused by what agent? A. Phenopthalein 23
  • 24. B. Naproxen C. Tetracycline D. Barbiturates E. Pseudoephedrine Correct Choice All agents listed are associated with fixed drug eruptions, however, pseudoephedrine hydrochloride is the one most commonly associated with non-pigmenting fixed drug eruptions 91) The Wolff-Chaikoff effect is associated with what medication? A. Potassium iodide Correct Choice B. Zidovudine C. Bexarotene D. Thalidomide E. Hydroxychloroquine The Wolff-Chaikoff effect is the inhibition of thyroid hormone synthesis from excess iodides which block organic iodides from binding in the thyroid. In patients with normal thyroid function, autoregulatory mechanisms allow for escape from this effect. In patients with impaired autoregulatory mechanisms, the Wolff-Chaikoff effect can lead to hypothyroidism. Thyroid function should be evaluated and monitored with patients started on potassium iodide 92) Which drug has been associated with an increased incidence of serum sickness in children? A. CefaclorCorrect Choice B. Clarithromycin C. Ciprofloxacin D. Clindamycin E. Rifampin Cefaclor has been associated with an increased incidence of serum sickness in children; the other drugs have not 93) Combination oral contraceptives decrease free testosterone levels by: A. Acting as competitive inhibitors of the androgen receptor B. Directly binding free testosterone C. Acting as a GnRH antagonist D. Acting as a GnRH agonist E. Increasing SHBG (sex hormone binding globulin) production Correct Choice Oral contraceptives decrease free testosterone levels by increasing the production of sex hormone binding globulin (SHBG). 94) Thalidomide is most associated with what adverse effect? A. Distal motor neuropathy B. Sensory neuropathy Correct Choice 24
  • 25. C. Oral ulceration D. Hypothyroidism E. Photosensitivity The most common presentation of the neuropathy from thalidomide is a mild proximal muscle weakness with symmetric painful paresthesias of the distal extremities with accompanying lower limb sensory loss. Hypothyroidism is a rarely reported adverse effect. 95) The mechanism of action of efalizumab in the treatment of psoriasis is: A. Inhibition of p-selectin B. Inhibition of tumor necrosis factor alpha C. Inhibition of T cell trafficking into the skin Correct Choice D. Inhibition of e-selectin E. Inhibition of macrophage maturation Efalizumab is a humanized monoclonal antibody directed against CD11a, a component of LFA1. Efalizumab blocks both T cell activation and trafficking of T cells to the skin. It is given once weekly as a subcutaneous injection. It’s side effects include rare thrombocytopenia and occasional rebound of psoriasis upon its abrupt discontinuation 96) All of the following retinoids are excreted in the urine EXCEPT: A. BexaroteneCorrect Choice B. Etretinate C. Isotretinoin D. Acitretin E. Tretinoin Bexarotene is excreted via hepatobiliary excretion. The others are excreted in bile and urine 97) The steroid with the least minerocorticoid activity is: A. Hydrocortisone B. Prednisone C. Methylprednisolone Correct Choice D. Cortisone E. Prednisolone Of the corticosteroids listed, the steroid with the lowest mineralcorticoid activity is methylprednisolone. Minerocorticoids act on the kidney to decrease the rate of sodium excretion (with accompanying retention of water). Triamcinolone, dexamethasone, and betamethasone also have low mineralcorticoid activity The steroid with the least minerocorticoid activity is: A. Hydrocortisone B. Prednisone C. Methylprednisolone Correct Choice 25
  • 26. D. Cortisone E. Prednisolone Of the corticosteroids listed, the steroid with the lowest mineralcorticoid activity is methylprednisolone. Minerocorticoids act on the kidney to decrease the rate of sodium excretion (with accompanying retention of water). Triamcinolone, dexamethasone, and betamethasone also have low mineralcorticoid activity 98) The most common side effect of treatment with interferon-alpha is: A. Liver toxicity B. Flu-like symptomsCorrect Choice C. Spastic diplegia D. Weight loss E. Nausea The most common side effect of treatment with interferon-alpha is flu-like symptoms of fever, chills, myalgias, headache and arthralgias. Prophylactic administration of non steroidal anti-inflammatory medications may alleviate some of these symptoms 99) Which of the following medications is most likely to interefere with the efficacy of oral contraceptives? A. RifampinCorrect Choice B. Doxycycline C. Tetracycline D. Amoxicillin E. Trimethoprim-sulfamethaxasole Rifampin has been shown to decrease the efficacy of oral contraceptives. It is an inducer of cytochrome p450 which increases the metabolism of hormones thereby decreasing the efficacy of oral contraceptives. There is no clear decrease in oral contraceptive efficacy with concomitant use of ampicillin, ciprofloxacin, clarithromycin, doxycyline, metronidzole, ofloxacin, or tetracycline 100) Which chemical sunscreen has UVB and UVA II absorption capability? A. Titanium dioxide B. Oxybenzone Correct Choice C. Methyl anthranilate D. Padimate O E. Octyl salicylate The benzophenones, oxybenzone and dioxybenzone, have the broadest absorption spectrum of the chemical sunscreens, with UVB and UVA II range. Methyl anthranilate, octyl salicylate, and padimate O are UVB-absorbing chemicals. Titanium dioxide is not a chemical absorber, it is a physical blocker 101) Alternate-day administration of oral steroids can reduce all of the following side effects except? 26
  • 27. A. Growth impairment B. Cataracts Correct Choice C. HPA axis suppression D. Peptic ulcer disease E. Opportunisitic infection Alternate-day corticosteroid dosing regimens does not decrease the risks of posterior subcapsular cataracts, osteoporosis, and possibly osteonecrosis 102) Which of the following medications can lead to hematologic toxicity when combined with methotrexate? A. Dapsone B. None of these answers are correct C. Sulfonamides D. All of these answers are correctCorrect Choice E. Trimethoprim All of the above inhibit the folic acid metabolic pathway, and can lead to hematologic toxicity when combined with methotrexate 103) Terbinafine exerts its antifungal activity by what manner? A. Interference with cell respiratory processes B. Inhibition of 14-a demethylase C. Inhibition of epoxide hydroxylase D. Inhibition of squalene epoxidase Correct Choice E. Direct binding to membrane sterols, increasing permeability Terbinafine, an allylamine, interferes with ergosterol synthesis by inhibiting squalene epoxidase. The azoles inhibit 14-a demethylase. Nystatin is a polyene which binds irreversibly to membrane sterols, resulting in a permeability shift. Ciclopirox does not appear to affect sterol biosynthesis but instead interferes with cell respiratory processes 104) What antifungal is known to cause gynecomastia and impotence? A. Ketoconazole Correct Choice B. Terbinafine C. Griseofulvin D. Itraconazole E. Fluconazole Ketoconazole is known to cause gynecomastia and impotence, by interfering with androgen and glucocorticoid synthesis 105) All of the following agents exert their function in a cell-cycle specific manner except: A. Hydroxyurea 27
  • 28. B. Methotrexate C. Azathioprine D. Cyclophosphamide Correct Choice E. 5-fluorouracil Cyclophosphamide is a cell-cycle nonspecific agent, which produces DNA cross-linkages at any point in the cell cycle. Methotrexate, azaithioprine, and hydroxyurea are S-phase specific cytotoxic agents. 5-fluorouracil is a cell-cycle specific pyrimidine antagonist 106) The t 1/2 of isotretinoin is: A. 7 hours B. 2 days C. 1 hour D. 120 days E. 20 hours Correct Choice The t 1/2 of isotretinoin is 20 hours. The other answers list the t 1/2 times of various retinoids 107) All of the following topical antioxidants have demonstrated cutaneous anticarcinogenic effects in mice except: A. ZincCorrect Choice B. Silymarin C. Vitamin E D. Vitamin C E. Tea polyphenois Anti-oxidants are thought to be protective against photoinjury by neutralizing oxygen radicals. Vitamin C, Vitamin E, tea polyphenois, and silymarin are all anti-oxidants 108) A 15 year old boy presents with a 4 month history of pigmented bands on several fingernails and toenails. The most like etiology is: A. Nevomelanocytic nevi B. Peutz-Jeghers syndrome C. Acral lentiginous melanoma D. Chloroquine therapy E. Minocycline therapy Correct Choice Melanonychia occurring simultaneously on several nails is most likely to be due to minocycline therapy. Blue-black pigmentation may be present in nails, skin, scars and sclerae 109) What is the treatment of choice for methemoglobinemia? A. Observation B. Aspirin 28
  • 29. C. Methylene blueCorrect Choice D. Iron E. Hydration Methylene blue is redued in the presnce of NADPH and diaphorase II to leukomethylene blue, which then reduces methemoglobin (Fe3+) to hemoglobin (Fe2+) 110) Which biologic agent is infused intravenously? A. Etanercept B. Alefacept C. InfliximabCorrect Choice D. Efalizumab E. None of these answers are correct Infliximab is infused intravenously 111) Which medication is the most likely cause of this drug eruption? A. Sulfur based medications B. Nonsteroidal anti-inflammatory medications C. Methotrexate D. Beta-lactam antibioticsCorrect Choice E. Calcium channel blockers Acute generalized exanthematous pustulosis (AGEP)is characterized by small pustules on erythematous base with evenutal desqumation. As opposed to most drug reactions, AGEP may occur within one week, 50% occur within the first 24 of exposure 112) The mechanism action of this cytotoxic agent is via inhibition of IMP dehydrogenase. A. Mycophenolate mofetilCorrect Choice B. Azathioprine C. Hydroxyurea D. Methotrexate E. 5-fluorouracil Mycophenolate mofetil (cellcept), a purine analog, blocks de novo purine synthesis by inhibiting the enzyme inosine monophosphate dehydrogenase 113) All of the following are reported cutaneous side effects of zidovudine EXCEPT: A. Trichomegaly B. Periungual pyogenic granulomas C. Diffuse and oral hyperpigmented macules D. None of the above (all are reported side effects)Correct Choice 29
  • 30. E. Hyperpigmented streaks in nails All of the following are cutaneous side effects reported with zidovudine, a nucleoside HIV reverse transcriptase inhibitor. Periungal/paronychial eruptions resulting in pyogenic granuloma-like lesions have also been reported with other HIV medications, including indinavir and lamivudine 114) Which of the following biologic therapies is pregnancy category C? A. Infliximab B. Alefacept C. Etanercept D. EfalizumabCorrect Choice E. Adalimumab All of the above drugs are pregnancy category B except efalizumab which is category C 115) Which of the following may cause an acneiform eruption? A. ACTH Correct Choice B. Fluoxetine C. Finasteride D. Valproic acid E. Methotrexate Many medications are associated with acneiform eruptions, including halogens (bromide and iodide), androgenic hormones such as testosterone, ACTH, corticosteroids, isoniazid (INH), lithium, phenytoin, and vitamins B2, B6 and B12 116) Which cell type is increased by glucocorticoids? A. Neutrophils Correct Choice B. Monocytes C. T-cells D. Eosinophils E. B-cells Glucocorticoids alter the balance of circulating leukocytes, causing an increase in the number of polymorphonuclear leukocytes and diminishing the numbers of lymphocytes, eosinophils, and monocytes 117) Over use of of which medication may lead to this clinical image? A. Topical antibiotic B. Topical steroid C. Calcipotriene D. Imiquimod E. HydroquinoneCorrect Choice 30
  • 31. Exogenous ochronosis has been reported with prolonged use of high concentration hydroquinone. On pathology, a characteristic ochre colored deposit is noted between the collagen bundles 118) Which of the following events is most important in the pathogenesis of this painful eruption? A. Increased expression of FasLCorrect Choice B. Reduction in circulating IL-6 C. Cleavage of desmoglein 1 D. Reduction in circulating tumor necrosis factor E. Overexpression of keratins 6 and 16 Toxic epidermal necrolysis is a life threatening drug eruption characterized by widespread epidermal necrosis. The exact etiology of the keratinocyte necrosis has not been fully elucidated. However, FasL (FasL and Fas are able to trigger apoptosis) has been shown to be upregulated in TEN 119) Which antiparasitic agent acts by inhibiting fumarate reductase? A. Cidofovir B. ThiabendazoleCorrect Choice C. Ivermectin D. Permethrin E. Lindane Thiabendazole inhibits fumarate reductase, a helminth-specific enzyme. It is used to treat creeping eruption or cutaneous larva migrans and larva currens. Ivermectin blocks glutamate-gated chloride ion channels, and is used to treat strongyloidiasis, onchocerciasis, and Norwegian scabies. Lindane is an organochloride which blocks neural transmission, and is effective against scabies, pubic lice, head lice, and body lice. Permethrin disables sodium transport channels in the nerve cell membrane of the parasite. Cidofovir is an antiviral nucleotide analogue 120) Which chemotherapeutic agent has been reported to cause acral sclerosis with Raynaud's phenomenon? A. BleomycinCorrect Choice B. Methotrexate C. Interferon D. Actinomycin E. 5-Fluorouracil Bleomycin is an antibiotic that induces single strand breaks in the DNA. Reactions to bleomycin include flagellae hyperpigmentation, acral sclerosis with Reynaud's, penile calcification, and a morbilliform eruption 121) Which of the following medications is most likely to have caused this reaction? A. Lithium B. Diclofenac C. Vancomycin 31
  • 32. D. Pseudoephedrine hydrochlorideCorrect Choice E. ACE inhibitor Fixed drug eruptions occur 30 mintues to 8 hours after ingestion of offending agent. After rechallenge with the same agent, the fixed drug eruption will recur. Potential causes of a fixed drug eruption include analgesics, sulfonamides, barbituates, pseudoephedrine and anticonvulsants 122) Which of the following chemotherapeutic agents has been linked to acneiform eruptions? A. CetuximabCorrect Choice B. Doxorubicin C. Cytarabine D. Bleomycin E. Cisplatin Cetuximab is a chimeric anti-epidermal growth factor receptor antibody that is FDA approved to treat advanced colorectal cancer. Acneiform eruptions have been reported to occur in up to 1/3 of patients 123) What family of medications is associated with xerosis? A. Sulfonylureas B. Cholesterol lowering agentsCorrect Choice C. Beta blockers D. Loop diuretics E. Calcium channel blockers Medications that alter the lipid composition of the epidermis and stratum corneum may impair the normal barrier function of the skin. Cholesterol lowering medications like HMG-CoA reductase inhibitors and niacin may cause xerosis through this mechanism 124) Which of the following are retinoid side effects? A. Reversible hyperthyroidism B. Delayed epiphyseal closure C. Diffuse interstitial skeletal hypo-ostosis D. All of these answers are corect E. Pseudotumor cerebriCorrect Choice Bexarotene has been shown to cause reversible hypothyroidism, not hyperthyroidism. Systemic retinoids have been shown to cause diffuse interstitial skeletal hyperostosis, premature epiphyseal closure, and pseudotumor cerebri (risk increased with concommitant use of tetracyclines 125) Which of the following is a potentially irreversible ocular side effects of antimalarial agents? A. Pterygium B. RetinopathyCorrect Choice C. Cataracts 32
  • 33. D. Neuromuscular eye toxicity E. Corneal deposition - causing halos, blurred vision, photophobia Three types of ocular adverse effects may develop from antimalarials: corneal deposits, neuromuscular eye toxicity and retinopathy. Only retinopathy is potentially irreversible. It is recommended that a patient be evaluated for retinopathy at baseline, then every 6 months by an ophthamologist. Testing visual acuity, visual fields and performing a funduscopic examination are considered acceptable for screening purposes 126) Clinical evidence of hypothyroidism can be induced by which drug? A. Isotretinoin B. Gold C. Bexarotene Correct Choice D. Griseofulvin E. Acitretin Bexarotene can cause reversible elevations in TSH levels and reductions in total T4 levels, associated with mild symptoms of hypothyroidism 127) Which of the following agents is NOT a UVB blocker? A. SulisobenzoneCorrect Choice B. Padimate A C. PABA D. Salicylates E. Cinnamates Sulisobenzone is a benzophenone, a UVA blocker. The others are UVB blockers. Padimate A is a PABA derivative 128) Which of the following drugs has been known to cause penile erosions? A. Penciclovir B. Abacavir C. Cidofovir D. Gancyclovir E. FoscarnetCorrect Choice Foscarnet has been reported to cause penile erosions 129) Peak vulnerability to thalidomide occurs between which days of gestation? A. Days 15-20 B. Days 37-56 C. Days 21-36Correct Choice D. Days 57-70 33
  • 34. E. Days 1-14 Peak vulnerability to thalidomide occurs between days 21 to 36 of gestation, during which only a single dose will cause birth defects to occur. Birth defects associated with thalidomide include phocomelia (underdevelopment of arms and legs, the most common birth defect), ear malformation, and gastrointestinal and urogenital defects 130) Extension of this blister by application of perpendicular pressure is an example of what? A. Gorlin's sign B. Hutchinson's sign C. Nikolsky sign D. Fitzpatrick sign E. Asboe-Hansen's signCorrect Choice Toxic epidermal necrolysis is a serious and potentially life threatening reaction to medications. The most commonly implicated medications include penicillins, NSAIDS, and anti-convulsants. Asboe- Hansen's sign results in extension of a blister with perpendicular pressure. Nikolsky sign is the separation of epidermis from the dermis by application of tangential mechanical pressure 131) Which of the following side effects has not been reported in association with intravenous immune globulin? A. Hypotension B. Headache C. Stevens-Johnson syndromeCorrect Choice D. Anaphylaxis E. Flushing IVIG is used to treat several diseases including graft versus host disease, connective tissue disease, and autoimmune bullous dermatoses. Adverse effects include infusion reactions (headache, flushing, chills, myalgia, wheezing, achycardia, lower back pain, nausea, or hypotension). Anaphylaxis occurs rarely. Disseminated intravascular coagulation, transient neutropenia, and aseptic meningitis syndrome has been reported. Cutaneous adverse effects include eczematous eruptions and alopecia 132) The form of erythromycin most likely to cause jaundice is: A. EstolateCorrect Choice B. Gluceptate C. Lactobionate D. Ethylsuccinate E. Stearate The form of erythromycin that most likely causes jaundice is estolate 133) Which of the following is known to induce lichen planus-like eruptions? A. Doxepin B. Minocycline 34
  • 35. C. Mercury D. Gold Correct Choice E. Dapsone Mucocutaneous side effects of gold include stomatitis, cheilitis, lichen planus- like eruptions, and pityriasis rosea-like eruptions 134) Which of the following medications is most likely to result in increased carbamazepine levels? A. Rifampin B. Minocycline C. Erythromycin Correct Choice D. Azithromycin E. TMP-SMX Eythromycin inhibits the hepatic cytochrome P450 system and can increase serum levels and potential toxicities of carbamazapene, theophylline, warfarin, digoxin, methylprednisolone 135) Which of the following statements regarding drug interactions is true? A. Tobacco induces P-450 enzymes B. All of the above are trueCorrect Choice C. Drugs that induce CYP3A enzymes may decrease levels of drugs which act as substrates for CYP3A D. CYP3A inhibitors may increase levels and cause toxicity of drugs metabolized by cytochrome P-450 E. Terbinafine is not metabolized by cytochrome P-450 The most relevant drug interactions in dermatology involve the hepatic biotransformation pathways catalyzed by the cytochrome P-450 isoenzymes from the subfamilies CYP3A3/4. Drugs that induce CYP3A enzymes may decrease levels of drugs which act as substrates for CYP3A. CYP3A inhibitors may increase levels and cause toxicity of drugs metabolized by cytochrome P-450. Terbinafine is not metabolized by cytochrome P-450, but by cytochrome 2B6 instead. Tobacco induces P-450 enzymes 136) The anti-viral agent used most often for acyclovir-resistent HSV and VZV infections is: A. Pencyclovir B. Valacyclovir C. Foscarnet Correct Choice D. Famcyclovir E. Gancyclovir Foscarnet does not require phosphorylaton for antiviral activity. Thereofre, it is achieve against viruses resistant to acyclovir, famcyclovir, or gancyclovir on the basis of altered-kinase activities 137) What tetracycline is not phototoxic? A. Doxycycline 35
  • 36. B. Tetracycline C. Minocycline Correct Choice D. Demeclocycline E. Oxytetracycline Minocycline is not phototoxic. Demeclocycline and doxycycline are the most phototoxic of all the tetracyclines. Onycholysis can accompany tetracycline-induced phototoxicity 138) Anemia, leg ulcers, poikilodermatous skin changes, hepatitis, renal toxicity, and acral erythema are most commonly associated with what medication? A. 5-fluorouracil B. Doxorubicin C. Methotrexate D. Hydroxyurea Correct Choice E. Cyclosporine The constellation of adverse effects is most closely associated with hydroxyurea 139) What drug can potentiate bone marrow suppression when used concomitantly with azathioprine? A. Salicylates B. NSAIDS C. Sulfonamides D. Allopurinol Correct Choice E. Phenytoin Concomitant allopurinol use, which inhibits xanthine oxidase, can lead to excess toxic purine analogs via increased metabolism of azaithioprine via the HGPRT pathway, causing bone marrow suppression 140) What is the mechanism utilized by the co-administration of probenicid to raise blood levels of penicillins in patients with infections that require high blood levels? A. Synergistic effect of probenicid with penicillins B. Displacement of plasma proteins C. Competitive inhibition of b-lactam binding sites D. Inhibition of cytochrome P-450 hepatic biotransformation system E. Prolongs the half-life of penicillins by decreasing renal tubular secretion Correct Choice Probenicid is co-administered with penicillin to prolong its half-life through decreased renal tubular secretion when higher blood levels are warranted 141) Which of the following statements regarding the ocular toxicities of the antimalarial drugs is NOT true? A. Chloroquine and hydroxychloroquine should not be given together because of an additive effect on 36
  • 37. retinotoxicity B. True retinopathy is associated with "bull's eye" pigment deposition, central scotoma, and diminished visual acuity C. Premaculopathy associated with changes in visual fields without visual loss is reversible if the antimalarial is discontinued D. The 4-aminoquinolones may have significant associated ocular toxicity E. Risk of retinopathy is greatest for quinacrine, followed by chloroquineCorrect Choice Risk of retinopathy is greatest with chloroquine and does not exist for quinacrine 142) Which of the following statements regarding podophyllin is NOT correct? A. It arrests cells in telophaseCorrect Choice B. It is derived from the May apple plant C. It binds tubulin D. None (all of these statements are true) E. It is contraindicated in pregnancy Podophyllin is a crude cytotoxic extract from the May apple plant. It is antimitotic, arresting cells in metaphase (not telophase) by binding to the protein tubulin. It may be teratogenic and should not be used in pregnancy 143) Which cutaneous side effect is a common complication of nitrogen mustard therapy? A. Telangiectasia B. Allergic contact dermatitisCorrect Choice C. Hyperpigmentation D. Bullous drug eruption E. Fixed drug eruption Topical nitrogen mustard, or mechlorethamine, is an antineoplastic agent which works via alkylation thereby inhibiting DNA synthesis. Allergic contact dermatitis occurs in two-thirds of patients who are treated with topical nitrogen mustard in aqueous solution, but occurs in less than 5% of patients treated with the ointment based preparation 144) Cefaclor has been associated with increased incidence of what in children? A. Transaminitis B. Anaphylaxis C. Generalized tonic-clonic seizures D. Mononucleosis-like syndrome E. Serum sickness reaction Correct Choice The use of cefaclor has been associated with an increased incidence of serum sickness in children 145) Which of the following oral agents has been effective in the treatment of Norwegian scabies? A. Metroniddazole 37
  • 38. B. IvermectinCorrect Choice C. Griseofulvin D. Mebendazole E. Thiabendazole Ivermectin (Stromectol) is an anti-helminthic agent currently FDA-approved for the treatment of strongyloides and onchocerciasis. Several publications have reported efficacy of this agent in the treatment of scabies and head lice. Due to its low rate of adverse effects, its high rate of effectiveness, and its ease of administration, some authors consider this agent to be the treatment of choice for scabies and head lice. The mechanism of action of ivermectin is blockade of glutamate- gated, chloride ion channels, with adverse effects on nerve and muscle resulting in paralysis and death of the helminth or mite. The drug has a very low affinity for mammalian chloride channels resulting in its relatively low toxicity. Adverse effects are rare and have been associated with accidental intoxication. It should be avoided when there is compromise of the blood-brain-barrier. Ivermectin is pregnancy category C. There are virtually no associated drug interactions with oral ivermectin therapy 146) Of the medications listed below, the safest to use during pregnancy is: A. NSAIDS B. Estrogens C. Erythromycin estolate D. Doxycycline E. Penicillin Correct Choice Erythromycin estolate is contraindicated in pregnancy because of the risk of cholestatic hepatitis. Other forms of erythromycin are safer for use in pregnancy. Tetracyclines are category D and estrogens are category X. NSAIDS may promote persistent fetal circulation or oligohydramnios 147) Which of the following antifungal agents is contraindicated in patients with a history of porphyria? A. Ketoconazole B. Terfinafine C. Itraconazole D. GriseofulvinCorrect Choice E. Fluconazole Griseofulvin has been reported as a potential exacerbator of acute intermittent porphyria, and thus is contraindicated in patients with a history of porphyria 148) Which of the following statements regarding dapsone and sulfapyridine is true? A. Sulfapyridine has a similar but often more severe side effect profile than dapsone B. Dapsone hypersenstivity syndrome is characterized by lymphocytosis C. They exert their anti-inflammatory actions by stimulating the myeloperoxidase activity of polymorphonuclear leukocytes D. None of these answers are correct (all are false)Correct Choice E. Concomittant administration of cimetidine has been shown to increase the risk of 38
  • 39. methemoglobinemia None of the above statements are true. Dapsone and sulfapyridine exert their anti-inflammatory actions by inhibiting the myeloperoxidase activity and chemotactic abilities of polymorphonuclear leukocytes. Dapsone hypersenstivity syndrome is characterized by eosinophilia, as well as a severe mononucleosis-like reaction, including fever, erythroderma, hepatitis, and even death. Sulfapyridine has a similar but often less severe side effect profile. Cimetidine has been shown to provide some protection against methemoglobin formation 149) Which of the following statements is true regarding the absorption of antibiotics? A. Tetracycline absorption is impaired by the ingestion of calcium products but not iron B. Doxycycline absorption is impaired by the ingestion of dairy products and calcium C. None of the above (all of the above statements are false)Correct Choice D. Minocycline absorption is not significantly impaired by the ingestion of calcium products but it should be taken on an empty stomach E. Fluoroquinolone absorption is not altered by antacids Antacids decrease the absorption of fluoroquinolones and should be taken at least 2 hours after the drug. Tetracycline absorption is impaired by the ingestion of dairy products, calcium, and iron or zinc salts. Minocycline and doxycycline absorption is not impaired by the ingestion of those products; they may be taken on an empty stomach or with food. Thus, all of the statements are false 150) Treatment with isotretinoin has been shown to cause increased colonization of the skin with which of the following organisms? A. Pseudomonas aeruginosa B. Staphylococcus aureusCorrect Choice C. Demodex folliculorum D. Pityrosporum orbiculare E. Streptococcus pyogenes Staphylococcus aureus colonization tends to correlate with isotretinoin-induced reduction in sebum production and may lead to infections. This complication may possibly be prevented with pulsed intranasal mupirocen therapy. There has been a report of staphylococcus endocarditis in a patient with underlying aortic insufficiency 151) This retinoid targets RXR receptors: A. Etretinate B. Isotretinoin C. Tretinoin D. Acitretin E. BexaroteneCorrect Choice Bexarotene targets RXR receptors. It is used in the treatment of mycosis fungoides refractory to conventional therapy. Side effects include central hypothyroidism and hyperlipidemia. These should be treated with synthroid, lipitor, and fenofibrate 152) Neutrophilic eccrine hidradenitis is a side effect of which therapeutic agent? 39
  • 40. A. Interferon-alpha B. Granulocyte colony stimulating factor C. Cytarabine Correct Choice D. Bieomycin E. Intravenous immune globulin Neutrophilic eccrine hidradenitis most commonly occurs in the setting of a patient with acute myelogenous leukemia being treated with cytarabine. Clinical manifestations include tender, erythematous macules, papuls and plaques on the trunk, neck and extremities which resolve within a few days. Histologically, this drug eruption is defined by the presence of dense neutrophilic infiltrate within and around eccrine glands, with necrosis of eccrine epithelial cells 153) Gray-green discoloration of the mid-portion of permanent teeth is a side effect of? A. Doxycycline B. Fluoroquinolones C. Minocycline Correct Choice D. Tetracycline E. clindamycin In contrast to tetracycline staining of the teeth, which occurs in childhood and produces a brown discoloration along the gingival third, minocycline stains the permanent teeth in adults, with a gray- green discoloration of the mid-portion of the tooth 154) Side effects of PUVA include all of the following except: A. Headache B. Nausea C. Pruritus D. NeutropeniaCorrect Choice E. Insomnia Side effects of psoralen with ultraviolet A light (PUVA) include side effects which are due to phototoxic effects including pruritus, photoonycholysis, friction blisters, ankle edema and hypertrichosis. In addition, there are adverse effects from methoxypsoralen including gastrointestinal and neurologic effects, hepatotoxicity and exanthems. Neutropenia is not a described side effect of PUVA 155) A patient taking daily prednisone is advised to switch to alternate day dosing to decrease the risk of: A. Adrenal crisisCorrect Choice B. Glaucoma C. Aseptic bone necrosis D. Cataracts E. Osteoporosis Long-term therapy with oral corticosteroids can result in numerous adverse effects, including elevated risks of glaucoma, cataracts, hypertension, diabetes, osteoporosis, adrenal axis 40
  • 41. suppression, and aseptic bone necrosis. Alternate-day dosing or oral corticosteroids lowers the rate of adrenal axis suppression. It is hypothesized that during the off day, cell mediated immunity, white blood cells subset levels, and potassium excretion are normalized while the anti-inflammatory benefits of the drug persist. Alternate-day corticosteroid therapy should be employed once adequate disease control has been attained with daily dosing. Of note, the risk of cataracts, osteoporosis, and other adverse effects of long-term corticosteroid use are not minimized with alternate-day dosing 156) Penile erosions are a reported side effect associated with which medication? A. Trimethoprim-sulfamethoxazole B. Azaithioprine C. Bleomycin D. Acyclovir E. Foscarnet Correct Choice None 157) Which of the following biologic agents is pregnancy category C? A. All of these answers are correct B. Etanercept C. Alefacept D. Infliximab E. EfalizumabCorrect Choice Efalizumab is pregnancy category C. The other drugs listed are pregnancy category B 158) Which of the following is NOT known to increase methotrexate levels? A. None of these answers are correct(all are known to increase methotrexate levels)Correct Choice B. Tetracyclines C. Salicylates D. NSAIDs E. Phenothiazines Tetracyclines, phenytoin, phenothiazines, chloramphenicol, NSAIDs, salicylates, and sulfonamides, among other drugs, can all increase methotrexate levels by displacement of plasma proteins 159) What is the best medication to lower isotretinoin induced hypertriglyceridemia? A. Cholestyramine B. GemfibrozilCorrect Choice C. All are equally effective D. Simvastatin E. Niacin Gemfibrozil generally reduces trygliceride levels to a greater extent than niacin, cholestyramine, and the HMG-CoA reductase inhibitors 41
  • 42. 160) Keratinocyte differentiation is enhanced by retinoids with all of the following EXCEPT: A. Increased keratohyalin granules B. Increased filaggrin production C. Stimulation of ornithine decarboxylaseCorrect Choice D. Odland body secretion of lipids E. Increased keratin filaments Keratinocyte differentiation is enhanced by retinoids with increased filaggrin production, increased keratohyalin granules, keratin filaments, and Odland body secretion of lipids. Retinoids directly inhibit ornithine decarboxylase and therefore lessen inflammatory hyperplasia 161) Which member of the tetracycline family is most likely to have caused this photomediated reaction? A. Doxycycline B. Oxytetracycline C. Minocycline D. DemeclocyclineCorrect Choice E. Tetracycline The tetracycline family of antibiotics are bacteriostatic and act by inhibiting protein synthesis. Each member of the family may cause photosensitivity, but demeclocycline is the most photosensitizing 162) The treatment of choice for erythema nodosum leprosum (ENL) is: A. Rifampin and clofazamine B. Clofazamine C. Rifampin D. Thalidomide Correct Choice E. Isoniazid, rifampin and clofazamine NEEDS EXPLANATIONS 163) The laboratory abnormality most associated with cyclosporine is: A. Hyperkalemia Correct Choice B. Hypermagnesemia C. Hypouricemia D. Increased LDH E. Hyponatremia The laboratory abnormalities associated with cyclosporine are decreased magnesium, increased potassium, and increased uric acid. Renal function and blood pressure must also be monitored closely in patients using cyclosporine 42
  • 43. 164) Concomitant use of methotrexate and what other drug is contraindicated because of the potential increased risk of pancytopenia? A. Tetracycline B. NSAIDs Correct Choice C. Systemic retinoids D. Folic acid E. Acetaminophen Drugs that simultaneously inhibit the folate metabolic pathway, such as NSAIDS, dapsone, or trimethoprim-sulfamethoxazole, can increase hematologic toxicity when combined with methotrexate 165) What is the treatment of choice for CMV retinitis? A. Ganciclovir B. Combovir C. FoscarnetCorrect Choice D. Cidofovir E. Acyclovir Foscarnet is indicated for the treatment of CMV retinitis and acyclovir resistant HSV, both of which can been seen in HIV patients. Foscarnet acts by inhibiting viral specific DNA polymerase. Unlike acyclovir, it does not require activation by thymidine kinase. Nephrotoxicity and seizures are major side effects 166) What is the most common cause of nonpigmented fixed drug eruption? A. Tetracycline B. Sulfonamides C. Naproxen D. Phenolphthalein E. PseudoephedrineCorrect Choice Pigmented incontinence is usually prominent in a fixed drug eruption; yet occasionally, fixed drug reactions do not result in long-lasting hyperpigmentation. The so-called nonpigmented fixed drug eruption is distictive. Pseudoephedrine hydrochloride is by far the most common cause 167) Which drug is the most likely to cause these changes in the gingiva? A. Doxycycline B. Acitretin C. Methotrexate D. CyclosporineCorrect Choice E. Cephalexin Gingival hyperplasia is a well described reaction to multiple medications including cyclosporine, phenytoin, and calcium channel blockers. All of these medications affect the influx of calcium into the cells which enhances the function of fibroblasts 43
  • 44. 168) The anti-CD 20 antibody rituximab is FDA-approved for treatment of which of the following? A. Paraneoplastic pemphigus B. Mycosis fungoids C. Non-Hodgkin’s lymphomaCorrect Choice D. Psoriasis E. Metastatic melanoma Rituximab (brand name Rituxan) is a monoclonal antibody is approved for the treatment of CD20 non-Hodgkin’s lymphoma. Rituximab is a monoclonal antibody directed against B lymphocytes which are CD20. 169) The combination of doxorubicin and which medications has been reported to cause sticky skin? A. Amphotercin B. G-CSF C. Vancomycin D. Cisplatin E. KetoconazoleCorrect Choice Polsen et. al. reported a 29% incidence of sticky skin in patients treated with high dose ketoconazole and doxorubicin for prostate cancer. Other medications reported to cause this include etretinate and tretinoin. Polsen JA, Cohen PR, Sella: Acquired cutaneous adherence in patients with androgen-independent prostate cancer receiving ketoconazole and doxorubicin: medication-induced sticky skin. J Am Acad Dermatol: 32 (4):571-5 1995 170) Which of the following cytotoxic agents has been associated with poikiloderma of the dorsal hands with a band-like distribution of the fingers and toes? A. Intralesional bleomycin B. HydroxyureaCorrect Choice C. Azathioprine D. Doxorubicin E. Flurouracil Hydroxyurea has been associated with poikiloderma of the dorsal hands with a band-like distribution of the fingers and toes. It has also been associated with diffuse hyperpigmentation, and with leg ulcers upon withdrawal 171) If a patient develops a dilantin hypersensitivity reaction, which anticonvulsant is the better alternative therapy? A. Phenobarbital B. None of these answers are correct 44
  • 45. C. Valproic acid Correct Choice D. Phenytoin E. Carbamezapine Carbamezapine, phenytoin, and phenobarbital are known to cross-react with one another 172) Hemorrhagic cystitis is a risk of what chemotherapy? A. Chlorambucil B. Hydroxyurea C. Vinca alkaloids D. Cyclophosphamide Correct Choice E. Azaithioprine Bladder toxicity is due to the acrolein metabolite of cyclophosphamide. Hemorrhagic cystitis is associated with the increased risk of transitional cell carcinoma of the bladder. Mesna has been used to reduce the toxic effect. The risk of cystitis is avoided by adequate fluid intake, frequent voiding, and careful screening for hematuria 173) All of the following are true regarding water-soluble retinoids EXCEPT: A. They include bexarotene B. They include etretinateCorrect Choice C. They include isotretinoin D. They are undetectable in the serum after 1 month of stopping therapy E. They have very little lipid deposition Isotretinoin, acitretin, and bexarotene are water-soluble, with very little lipid deposition. Etretinate is 50 times more lipophilic than acitretin, with increased storage in adipose tissue The anti-HIV medication best known for causing a severe reaction which can result in fatality upon rechallenge is: A. Zidovudine B. Indinavir C. Nevirapine D. Abacavir Correct Choice E. Didanosine The hypersensivity reaction associated with abacavir usually resolves with cessation of the drug, however upon rechallenge the reaction can be life-threatening 45